Anda di halaman 1dari 133

BRILLIANT PUBLIC SCHOOL,

SITAMARHI
(Affiliated up to +2 level to C.B.S.E., New Delhi)

Class IX
Mental Ability Reasoning
N.T.S.E. & Olympiad Study Package

Session: 2014-15

Office: Rajopatti, Dumra Road, Sitamarhi (Bihar), Pin-843301


Ph.06226-252314 , Mobile:9431636758, 9931610902
Website: www.brilliantpublicschool.com; E-mail: brilliantpublic@yahoo.com
Subject : Mental Ability Class : IX (C.E.)

CONTENT
PART - II
S.No. Topics Page No.
1. Series Completion 1 - 14

2. Coding - Decoding 15 - 20

3. Alphabet test & Number Ranking 21 - 26

4. Mathematical Operations 27 - 34

5. Puzzle Test 35 - 42

6. Seating Arrangement 43 - 48

7. Blood Relations 49 - 54

8. Direction Sense Test 55 - 62

9. Analogy 63 - 74

10. Classification 75 - 78

11. Logical Venn Diagram 79 - 86

12. The Calendar 87 . 92

13. The Clock Test 93 - 98

14. Cube Test 99 - 106

15. The Dives 107 - 114

16. Figure Partition 115 . 120

17. Dost Situation 121 - 126

18. Mirror Image and Water Image 127 - 132

19. Non - Verbal Series 133 . 142

20. Non - Verbal Analogy & Classification 143 - 150

21. Pyramids & Miscellaneous 151 – 158


SERIES COMPLETION
Series completion problems deals with number, alphabets and both together. While attempting to
solve the question, you have to check the pattern of the series. Series moves with certain
mathematical operations. You have to check the pattern.
Type of questions asked in the examination :
(i) Find the missing term(s).
(ii) Find the wrong term(s).
NUMBER SERIES :
(a) Some Important Patterns :
(i) a, a ± d, a ± 2d, a ± 3d, .............. (Arithmetic Progression)

(ii) a, ak ak2, ak3, ......................(Geometric Progression)


a a a
(iii) a, , , , …………....... (Geometric Progression)
k k2 k3
(iv) Series of prime number - i.e. 2, 3, 5, 7, 11 .......
(v) Series of composite numbers - i,e, 4, 6, 8, 9, 10, 12, ........
Direction : (1 to 8) Find the missing numbers :
Ex.1 21, 24, 27, 30, ?
Sol. As per series, a, a + d, a + 2d, ...... a = 21, d = 3 a + 4d = 21 + 4 × 3 = 33Ans. 33
Ex.2 9, 18, 36, ? 144

Sol. As per series a, ak, ak2, ak3, ....... a = 9+, k = 2 ak3 = 9 × 23 = 72 Ans. 72
Ex.3 2, 6, 14, 26, ?
Sol. The pattern is + 4, +8, +12, +16, .......Ans. 42
Ex.4 1, 4, 12, 30, ?
Sol. Each term is equal to the previous term multiplied by 2 and 2, 4, 6, ..... are added to the products
respectively. Hence, the next term = 30 × 2 + 8 = 68.Ans. 68
Ex.5 8, 12, 21, 46, 95, ?

Sol. The pattern is + 22, + 32, 52, + 72, .......... ∴missing number = 95 + 112 = 216Ans. 216
Ex.6 3, 9, 36, 180, ?
Sol. Each term is multiplied by 3, 4, 5 and so on respectively. Therefore, the next term would be
180 × 6 = 1080.Ans. 1080
(b) Multiple Series :
A multiple series is a mixture of more than one series :
Ex.7 4, 7, 3, 6, 2, 5, ?
Sol. The sequence is a combination of two series.

1
I 4, 3, 2, ?
II 7, 6, 5
The pattern followed in I is -1, -1, -1
∴ missing number = 2 - 1 = 1Ans. 1
Ex. 8 14, 15, 12, 16, 9, 18, 4, 21, ?
Sol. The sequence is a combination of two series.
I 14, 12, 9, 4, (.....) and
II 15, 16, 18, 21
The pattern followed in I is -2, -3, -5, ..... ∴ missing number = 4 - 7 = - 3Ans. -3
Direction : (9 to 10) Find the wrong term(s) -
Ex.9 9, 13, 21, 37, 69, 132, 261
Sol.

Hence, the wrong number is 132 and should be replaced by 133.Ans. 132
Ex.10 5, 8, 10, 12, 15, 18, 20, 23
Sol.

Therefore, number 12 is wrong and should be replaced by 13.Ans. 12


ALPHABET SERIES (SERIES OF LETTERS) :
(a) Pattern of Alphabets Show Variation Based on :
(i) Position of the letters (ii) Difference between the alphabets
(i) Position of alphabets :
Alphabets in order :
ABCD E FGHI J K L M N O P Q R S T U V W X Y Z
1 2 3 4 5 6 7 8 9 10 11 12 13 14 15 16 17 18 19 20 21 22 23 24 25 26
Alphabets in reverse order :
Z YX WV UT SR Q P O N M L K J I H G F E D C B A
1 2 3 4 5 6 7 8 9 10 11 12 13 14 15 16 17 18 19 20 21 22 23 24 25 26
Direction : (11 to 20) Find the missing term :
Ex 11. B, D, G, I, ?, N
Sol. Gap of letters between the two consecutive terms is increased by +1. So, the missing would be L.
Ex.12 A, Y, D, W, G, U, J, ?
Sol. The given sequence consists of two series :
I A, D, G, J in which each letter is moved three steps forward to obtain the next term
II Y, W, U, ? in which each letter is moved two steps backward to obtain the next term.
So, the missing term would be S.

2
Ex .13 H, L, P, T, X, ?
Sol.

As the difference between alphabetical positions is constant, the next term would be having
alphabetical positions 28, i.e. 26 + 2 = B. So, the missing term would be B.

Ex.14 AG, LR, WC, HN, ?


Sol. The first letter of each group and the second letter of each group differs by 11 letters between
them.

Therefore, the next group of letter would be SY.

Ex.15 HEJ, JGL, LIN, NKP, ?


Sol. First letter of each group differs by 2 letters. Second letter of each group differs by 2 letters. Third
letter of each group differs by 2 letters. All the letters differ in the forward direction. Hence, the
next choice would be PMR.
Ex.16 YAL, TCP, OET, JGX, ?
Sol. First letter of each group differs by 5 letters in the backward direction. Second letter of each group
differs by 2 letters in the forward direction. Third letter of each group differ by 4 letters in the
forward direction. Hence, the next choice would be EIB.
Ex.17 AD, EI, JO, PV, ?
Sol. The first letter of subsequent groups have a difference of 4, 5 and 6 places respectively, whereas
the second letter of the subsequent groups has a difference of 5, 6 and 7 places respectively.
Therefore, on following the same patter, we get ‘WD’ as the nest term which would replace the
question mark.
Ex.18 Find the term which would replace the questions mark ?
XYQ, ZAR, BCS, DET, ?
Sol. Here, first two terms of every group of letters are in continuation, like XY, ZA, BC, DE, and the
third letter of each group is again in forward continuation, i.e. Q, R, S, T. Hence, the term
replacing the question mark would be FGU.

3
Ex.19 1 BR, 2 EO, 6 HL, 15 KI, ?
Sol. The first number in the terms follow the sequence +12, +22, +32, +42, ........ The second letter of
each group differs by 3 letters in the forward direction. Third letter of each group differs by 3
letters in the backward direction. Hence, the next choice would be 31NF.
Ex.20 Z, L, X, J, V, H, T, F, ?, ?

Sol.

Directioins : (21 to 22) Find the wrong term(s) :

Ex.21 DOU, EPV, FQW, GRX, HTY, ITZ


Sol. In every term first second and third letter is in alphabetical order to its next term respectively.
Fourth term is not following the same rule. Hence, HTY is the wrong and should be replaced by
HXY.

Ex.22 D4V, G10T, J20R, M43P, P90N


Sol. First letter of every term is moved three forward in each next term. Second number of every term
of the pattern ⇒ × + 2 + 1, × 2 + 2, × 2 + 3, ....... and third letter of every term is moved two steps
backward. Hence G10T is the wrong term and should be replaced by G9T.

LETTER REPEATING SERIES :


Pattern of such questions is that some letters is sequence are missing.
(i) The letters may be in cyclic order (clockwise or anti-clockwise).
(ii) To solve a problem, we have to select one of the alternative from t he given alternatives. The
alternative which gives a sequence form of letters is the choice.

directions : (23 to 28) Find the missing term(s) :

Ex.23 a a_b a a_b b b_a


(A) baa (B) abb (C) bab (D) aab
Sol. we proceed step by step to solve the above series :
Steps :
1. The first blank space should be filled in by ‘b’ so that we have two ‘a’s followed by b’ s
2. Second blank space should be filled in by ‘a’ so that we have three a’s followed by three b’ s
3. The last blank space must be filled in by ‘a’ to keep the series in sequence

Ans. (A)

4
Ex.24 a_cab_a_c_b c
(A) bbac (B) abab (C) abba (D) bcba

Sol.

Series is abc/abc/abc/abc. So, pattern ab is repeated.

Ans. (D) bcba

Ex.25 a_abb_aa_ba_a_b
(A) ababa (B) aabba (C) aabab (D) aaabb
Sol. Series is aaabb/aaabb/aaabb. So, pattern aaabb is repeated.
Ans. (C) aabab

26. ba_cb_b_bab_
(A) acbb (B) bcaa (c) cabb (D) bacc
Sol. The series is b a b c / b a b c / b a b c
So, pattern babc is repeated
Ans. (D) bacc

Ex.27 a_bc_a_bcda_ccd_bcd_
(A) abddbd (B) acbdbb (C) adbbad (D) bbbddd
Sol. Series is aabcd/abbcd/abccd/abcdd
Ans. (C) adbbad

28. bc_b_c_b_ccb
(A) cbcb (B) bbcb (C) abbc (D) bcbc
Sol. (A) Series is bccb / bccb. So, pattern bccb is repeated
Ans. (A) cbcb

Directions : (29) given below are based on the letter series, In each of these series, some letters
are missing. Select the correct alternative. If more than five letters are missing, select
the last five letters of the series.

Ex.29 abcd_bc_e_ _de_ _ _ _ _ _


(A) deabc (B) edcba (C) decba (D) edabc
Sol. The series is a b c d / b c d e a / c d e a b / b e a b c
Thus the letters are written in a cyclic order.

Ans. (A) deabc

5
Direction : (30) There is a letter in the first row and a number series in the second row. Each
number in the number series stands for a letter in the letter series. Since in each of that
series some term are missing you have to find out as to what those terms are, and answer
the questions based on these as given below in the series.

Ex.30 _m y e _ _ y l x _ y l m _ _ l _ _ _
46_586___57_658____
The last five terms of the number series are
(A) 46758 (B) 74658 (C) 76485 (D) 4675
Sol. By taking e = 5, I = 4, m = 6, y = 7 and x = 8 the number series runs as 46758 67485 74658
46785. By taking the digits in the groups of five, we find that first digit of the first group (i.e. 4) is
the third digit of the second group and the last two digits have interchanged their positions. The
same rule applies in others groups also.
Ans. (D) 46785

MISSING TERMS IN FIGURES :

Directions : (31 to 40) Find the missing number(s) :

Ex.31 6 9 15
8 12 20
4 6 ?

(A) 5 (B) 10 (C) 15 (D) 231


Sol. In the first row, 6 + 9 = 15
In the second row, 8 + 12 = 20
∴ In the third row, missing number = 4 + 6 = 10.
Ans. (B) 10
Ex.32 3C 27D 9E
71 21K 3M
4D ? 7J
(A) 11E (B) 28G (C) 35 (D) 48F

Sol. The letters in the first row form a series C,D,E (a series consecutive letters). The letter in the
second row form a series I, K, M (a series of alternate letters). Similarly, the letters in the third row
will form the series D, G, J (a series in which each letter is there steps ahead of the previous one).
So, the missing letter is G. Also, the number in the second column is equal to the product of the
numbers in the first and third columns. So, missing number is (4 + 7) i.e. 28 Thus, the answer is
28 G.
Ans. (B) 28G

6
Ex.33

(A) 16 (B) 9 (C) 85 (D) 112


Sol. Hint : 42 + 52 = 16 + 25 = 41

12 + 22 = 1 + 4 = 5

62 + 7 = 36 + 49 = 85
Ans. (C) 85

Ex.34

(A) 38 (B) 64 (C) 4 (D) 16


Sol. The number ‘143’ given inside the triangle is the combination of 1, 16 , 9 . In same manner

number ‘236’ is combination of 4 , 9 , 36 . Thus, the answer is ‘4’.

Ans. (C) 4

Ex.35

(A) 127 (B) 142 (C) 158 (D) 198


Sol. In first figure, (101 + 15) - (35 + 43) = 116 - 78 = 38.
The same pattern would be followed in second figure.
∴ Missing number = (48 + 184) - (56 + 34) = 232 - 90 = 142.
Ans. (B) 142

Ex.36

(A) 18 (B) 12 (C) 9 (D) 6


Sol. The number at the bottom are the H.C.F. of above given numbers
Clearly 9, is the HCF of 36, 18 and 27.
Ans. (C) 9

7
Ex.37

(A) 16 (B) 21 (C) 61 (D) 81


14 18
Sol. In first figure 12 × = 84 . In second figure 9 × = 81
2 2
x 88 × 2
Let the missing number In third figure be x. Then 11× = 88 or x = = 16.
2 11
Ans. (A) 16

Ex.38

(A) 60 (B) 50 (C) 21 (D) 25


Sol. The square of the number at the bottom is equal to the product of the two upper numbers. Thus,

In first figure, 4 × 9 = 62 = 36. In second figure, 9 × 16 = 122 = 144.


400
Let the missing number in third figure be x. Then, 16 × x = 202 = 400 or x = = 25.
16
Ans. (D) 25

Ex.39

(A) 47 (B) 45 (C) 37 (D)


Sol. In first figure, 6 × 3 + 3 × 5 = 33
In second figure, 5 × 4 + 4 × 7 = 48
∴ In third figure, 5 × 4 + 3 × 5 = 35
Ans. (D) 35

Ex.40.

(A) 48 (B) 72 (C) 35 (D) 120


Sol. In first figure (15 - 5 ) × (2 + 6 ) = 80.
In second figure (9 - 4) × (7 + 6) = 85.
∴ In third figure, missing number = (13 - 11) × (16 + 8) = 48.

Ans. (A) 48

8
PRACTICE EXERCISE

Directions : (1 to 13) Find the missing term(s) -

1. 101, 100, ?, 87, 71, 46.


(A) 92 (B) 88 (C) 89 (D) 96
2. 100, 50, 52, 26, 28, ?, 16, 8.
(A) 30 (B) 36 (C) 14 (D) 32
3. 6, 24, 60, 120, 210, 336, ?, 720
(A) 496 (B) 502 (C) 504 (D) 498
4. 3, 1, 4, 5, 9, 14, 23, ?
(A) 32 (B) 37 (C) 41 (D) 28
5. 3, 6, 18, 72, 360, ?
(A) 720 (B) 1080 (C) 1600 (D) 2160
6. 78, 79, 81, ?, 92, 103, 119
(A) 88 (B) 85 (C) 84 (D) 83
7. 0, 6, 20, 42, 72, ?
(A) 106 (B) 112 (C) 110 (D) 108
8. 2, 9, 28, 65, ?
(A) 121 (B) 195 (C) 126 (D) 103
9. 1, 11, ?, 11, 11, 16, 11
(A) 1 (B) 11 (C) 6 (D) 192
10. 137, 248, 389, 470, ?
(A) 582 (B) 581 (C) 571 (D) 481
11. 3, 15, 35, ?, 99, 143
(A) 63 (B) 77 (C) 69 (D) 81
12. 9, 16, 30, 58, ?
(A) 104 (B) 114 (C) 116 (D) 118
13. 3, 12, 27, 48, 75, 108, ?
(A) 192 (B) 183 (C) 162 (D) 147

Directions : (14 to 16) Find the wrong term(s)


14. 2 5 1 20 30 47 65
(A) 5 (B) 20 (C) 30 (D) 47
15. 121, 143, 165, 186, 209
(A) 143 (B) 165 (C) 186 (D) 209

9
16. 9, 15, 24, 34, 51, 69, 90
(A) 15 (B) 24 (C) 34 (D) 51

Directions : (17 to 28) Find the missing term(s) -

17. X, U, S, P, N, K, I, ?
(A) J (B) K (C) M (D) F
18. Z, X, U, Q, L, ?
(A) F (B) K (C) G (D) E
19. A, H, N, S, W, ?
(A) A (B) Y (C) B (D) Z
20. Q, T, V, Y, A, ?
(A) B (B) C (C) D (D) F
21. X, A, D, G, J, ?
(A) N (B) O (C) M (D) P
22. AZ, YB, CX, WD, ?
(A) VE (B) UE (C) EU (D) EV
23. ZSD, YTC, XUB, WVA, ?
(A) HDC (B) CHI (C) HCD (D) DIC
24. RML, VIJ, ZFH, DDF, ?
(A) HDC (B) CHI (C) HCD (D) DIC
25. LRX, DJP, VBH, NTZ, ?
(A) ELS (B) FMR (C) GKS (D) FLR
26. P3C R5F T8I V12L ?
(A) Y170 (B) X17M (C) X170 (D) X160
27. MAD, OBE, SCH, YDM, ?
(A) HET (B) HES (C) GET (D) UAE
28. X 15 A, W 13 C, ? , 9 G, N 7 I
(a) T 12 E (B) R 11F (C) T 11E (D) R 13 D

Directions : (29 to 34 which sequence of letters when placed at the blanks one after the other will
complete the given letter series ?
29. a_b a a_a a _ _ ab
(A) a a a a (B) b a a a (C) b b a a (D) a b b a
30. a_ b a a _ b a a_b a
(A) a a b (B) b a b (C) b b a (D) b b b
31. _ a a b b_a_ a b_b
(A) b b a a (B) b a b a (C) b a a b (D) a b a b

10
32. b a b b b_b_b_b b
(A) b b a (B) b a b (C) a b a (D) a a a
33. a a b_a a a_b b a_
(A) b a a (B) a b b (C) b a b (D) a a b
34. m_l_ml_m_llm
(A) lmmm (B) lmlm (C) lmml (D) mllm

Direction : (35 to 36) given below are based on the leter sires, In each of these series, some letters
are missing. Select the correct alternative. If more than five letters are missing, select the
last five letters of the series.
35. x _ xxy _ x _ xy _ yxx _ _ yy _ y
(A) xyyy (B) xxyyx (C) yxxyx (D) xyxyx
36. _ _ r + tprptsrpst _ _ _ _
(A) pqrts (B) pqtrs (C) pqrst (D) qrpst

Directions : (37 to 38) There is a letter series in the first row and a number series in the second
row. Each number in the number series stands for a letter in the letter series. Since in each
of these series some term are missing you have to find out as to what those terms are, and
answer the questions based on these as given below in the series.
37. n_gf_t_fhtn__t_b_f
13_2450_4__3______
The last five terms of the number series are
(A) 50123 (B) 40331 (C) 40231 (D) 51302
38. _miax_irxa__ma______
4_5_73____6_______
The last five term of the letter series are
(A) r m x i a (B) x m r a i (C) x r m a I (D) r m i x a

Directions : (39 to 53) Find the missing terms in the given figures :
39.
1 7 6
3 3 ?
5 4 8
35 74 104

(A) 1 (B) 2 (C) 3 (D) 4

40.

(A) 33 (B) 145 (C) 135 (D) 18

11
41.

(A) 28 (B) 36 (C) 81 (D) 49

42.

(A) 16 (B) 14 (C) 20 (D) 22

43.

(A) 117 (B) 36 (C) 32 (D) 26

44.

(A) 26 (B) 25 (C) 27 (D) 30

45.

(A) 48 (B) 9 (C) 44 (D) 64

46.

(A) 25 (B) 129 (C) 7 (D) 49

47.

(A) 78 (B) 82 (C) 94 (D) 86

12
48.

(A) 14 (B) 18 (C) 11 (D) 26

49.

(A) 9 (B) 11 (C) 1 (D) 12

50. BIG - 792 HCA - 138 FED - 456 E?H 87?

(A) G, 6 (B) I, 9 (C) G, 5 (D) I, 5

51.

(A) 19 (B) 23 (C) 25 (D) 31

52.

(A) 3 (B) 4 (C) 5 (D) 6

53.

(A) 69 (B) 49 (C) 50 (D) 60

ANSWERS

Que. 1 2 3 4 5 6 7 8 9 10 11 12 13 14 15
Ans. D C C B D B C C C B A B D C C
Que. 16 17 18 19 20 21 22 23 24 25 26 27 28 29 30
Ans. C D A D C C D C C D C C C D D
Que. 31 32 33 34 35 36 37 38 39 40 41 42 43 44 45
Ans. D C A B A A D D B C A B B A C
Que. 46 47 48 49 50 51 52 53
Ans. A D D A C D B A

13
CODDING - DECODING

CODING-DECODING :
Coding is a method of sending a message to the receiver, such that the third person doesn’t
know about it. Code language is formed by certain rules & rules & patterns. To know this
language following certain rules is called ‘Decoding’.

TYPES OF ODING-DECODING :
(i) letter-letter coding
(ii) letter-number coding
(iii) To code letter/words in puzzle form
(iv) To code some objects in puzzle form
(v) To code as per table form/column form

Ex.1 If in any code language VOYAGE is coded as FIBAPX how is CROWN coded in that language
(A) OXPTD (B) DTPYO (C) OYPTD (D) DTPXO

Sol.

Similarly,

The pattern of letters is +1, +2, +1, +2 ...... & they are reversed.
Ans. (C)

Ex.2 In a certain code, MONKEY is written as XDJMNL. How is TIGER written in that code ?
(A) QDFHS (B) SDFHS (C) SHFDQ (D) UJHFS

Sol.

Ans. (A)

14
Ex.3 In a certain code HEATER is written as KBDQHO, how will you encode COOLER ?
(A) ALRIHV (B) FLRIHO (C) FLIRHO (D) None of these

Sol. So,

Ex.4 If JUMP is coded as KVLO then GIVE will be coded as :


(A) HHWD (B) HIUE (C) HJUD (D) HJVD

Sol. Similarly

Ans. (C)

Ex.5 if D = 4 and COVER = 63, then BASIS = ?


(A) 49 (B) 50 (C) 54 (D) 55
Sol. In the code A = 1, B = 2, C = 3, ......, X = 24, Y = 25, Z = 26
C O V E R So B A S I S

3 + 15 + 22 + 5 + 18 = 63 2 + 1 + 19 + 9 + 19 = 50
Ans. (B)
Ex.6 If cloud is called white, white is called rain, rain is called green, green is called air, air is called
blue, and blue is called water, where do the birds fly is ?
(A) Air (B) Cloud (C) White (D) Blue
Sol. Birds fly in air and air is called blue.
Ans. (D)
Ex.7 If room is called house, house is called roof, roof is called floor, floor is called terrace, where
will a person sand on ?
(A) Floor (B) Terrace (C) Roof (D) Room
Sol. A person stands on a floor and floor is called terrace.
Ans. (B)
Ex.8 In a certain language, tir me sac means green and tasty, dic sac for means tomato is green,
voc tir for means food is tasty. Which of the following does mean tomato is tasty in that code ?
(A) for tir dic (B) dic for sac (C) tir me for (D) None of these
Sol. In the first and second statements, the common code word is green and the common word is sac.
So, green means sac. In the first and third statements, the common code word is tasty and the
common word is tir. So, tasty means tir. Thus, in second and above statements tomato is tasty
means for tir dic.
Ans. (A)

15
Ex.9 In a certain code, 256 means red colour chalk, 589 means green colour flower, and 245
means white colour chalk. What digit in that code does mean white ?
(A) 2 (B) 4 (C) 5 (D) cannot be determined
Sol. In the first and second statements, the common digit is 5 and the common word is colour. So, 5
means colour. In the first and third statements, the common code digit is 2 and the common word
is chalk. So, 2 means chalk. Thus, is third and above statements 4 means white.
Ans. (B)

Directions : (10 to 13) In the following questions, two columns I and II have been given. In column
I few words are given and in column II their codes have been given using a particular rule.
The order of the smaller letter have been placed in jumbled up form. You have to decode
the language and choose the alternative whish is equal to letter asked in the question.
Column I Column II
(i) DESIGN (a) uklbjz
(ii) INFORM (b) cbxkqy
(iii) MOTHER (c) ygzwxc
(iv) RIGHTS (d) bjucgw
(v) TAILOR (e) wcpybv
(vi) GARDEN (f) vzcjlk

Ex.10 What is the code for letter N ?


(A) u (B) k (C) c (D) g
Sol. In statement (i) and (ii), common letters and I and N and common codes are b and k. Hence, it is
clear that IN stand for bk but not respectively. From statement (iv), it is clear that the world has
letter N and code k in its coding. Hence, code for N is k.
Ans. (B)
Ex.11 What is the code for letter F ?
(A) l (B) b (C) q (D) g
Sol. In the statement (ii), it is clear that world has letter F in it, which is not contained by an other word.
Similarly, its code has letter q, which is not contained by any other code. Hence, F stands for q.
Ans. (C)
Ex.12 What is the code for letter O ?
(A) y (B) k (C) v (D) c
Sol. From statements (iii) and (vi), it is clear that TOR ywc. From statement (ii), OR = yc. From
statement, (vi) R = c. Hence, O = y.
Ans. (A)

16
Ex.13 What is the code for letter S ?
A() z (B) w (C) u (D) x
Sol. From statements (i) and (iv), it is clear that STG = ubj. We have already found that I = b.d
Therefore, SG = uj. Now, from statement (vi) G = j, therefore S = u.
Ans. (C)

Directions : (14 to 15) In each questions there is a word written in capital letters with one letter
underlined. For each letter in that word there is a code written is small letters. That code is
denoted by either (A), (B), (C), (D) or (E) not in the same order. You have to find out the
exact code for the underlined letter in the word. The number of that code is the answer.
Please note that the same letter appearing in other word(s) may be coded differently.

Ex.14 PRISM
(A) r (B) 0 (C) h (D) q (E) 
Sol. P9-1) is o, R(-1) is q, I(-1) is h S(-1) is r and M(-1) is  .
Sol. (A)

Ex.15 WHICH
(A) f (B) g (C) u (D) e (E) j
Sol. W (-2) is u, H(+2) is J, I(-2) is g C(+2) is e and H(-2) is f.
Ans. (A)

PRACTICE EXERCISE

1. If TRAIN IS CODED AS RPYGL, the code for SCOOTER would be


(A) QAMMRCP (B) QBNNRCP (C) QAMMSBP (D) QBNNSBP

2. If SCIENCE is coded as UFJTJM, GENE will be coded as :


(A) HGQI (B) IHRJ (C) IHRI (D) IHSJ

3. If EQOKYO stands for DOLLAR and QQXMBP stands for POUNDS, then MARKET stands for :
(A) NYOLGW (B) NYOGLW (C) LYOLGW (D) NYOLWG

4. If in a certain code MANISH is written as NZMRHS, then how will RNJITA be written in the same
code ?
(A) IZMQRGZ (B) IZMPRGZ (C) IZMQRHZ (D) IZMQRIZ

5. If GOOD is written HQRH, how will you write DREAM ?


(A) ESPBN (B) ETHER (C) ETHPQ (D) ESHDR

17
6. If TRANSFER is coded as RTNAFSRE, then ELEPHANT would be coded as
(A) LEPEHATN (B) LEPEAHTN (C) LEEPAHTN (D) LEPEAHNT
7. In a certain code, PAINTER is written NCGPRGP, then REASON would be written as
(A) PCYQMN (B) PGYQMN (C) PGYUMP (D) PGYUPM
8. If BOOK is coded as 43, what will be the code number for PEN ?
(A) 53 (B) 33 (C) 35 (D) 43
9. In a certain code KAMAL is written as 29894, VIJAY is written as 35196 then the word VIMAL will
be coded as
(A) 29196 (B) 35894 (C) 35194 (D) 35196
10. If TOWER is coded as 81, what will be the code number for POWER ?
(A) 75 (B) 55 (C) 18 (D) 77
11. If MAN is coded as 28, what will be the code number for CHIKD ?
(A) 25 (B) 36 (C) 49 (D) 64
12. If CAR is 22 then SCOOTER = ?
(A) 33 (B) 44 (C) 11 (D) 95
13. If Eye is called Hand, Hand is called Mouth, Mouth is called Ear, Ear is called Nose and Nose is
called Tongue, with which of the following would a person hear ?
(A) Eye (B) Mouth (C) Nose (D) Ear
14. If orange is called butter, butter is called soap, soap is called ink, ink is called honey and
honey is called orange, Which of the following is used for washing clothes ?
(A) Honey (B) Butter (C) Orange (D) Ink
15. In a certain code, 256 means boys are good, 637 means amar is good, and 348 means lata is
bad. Which digit means amar in that code ?
(A) 2 (B) 7 (C) 6 (D) 8

Directions : (16 to 21) The following questions are based on the pattern as used for previous
questions. Understand the coding pattern and answer the questions.
Column I Column II
(i) FAMOUS (a) jcphxp
(ii) SATIRE (b) hqdbyn
(iii) FRIGHT (c) ybcnke
(IV) TANGLE (d) zewhnd
(V) ROVING (e) epbmyw
(VI) HUNTER (f) wdnbxk
16. What is the code used for the letter M ?
(A) q (B) x (C) j (D) e

18
17. What is the code used for the letter E ?
(A) d (B) n (C) b (D) k

18. What is the code used for the letter F ?


(A) w (B) p (C) d (D) c

19. What is the code used for the letter A ?


(A) h (B) q (C) b (D) n

20. What is the code used for the letter N ?


(A) e (B) p (C) m (D) w

21. What is the code used for the letter U ?


(A) d (B) n (C) b (D) x

Directions : (22 to 24) In each Q. there is a word written in capital letters with one letter
underlined. For each letter in that word there is a code written in small letters. That code is
denoted by either (A), (B), (C), (D) or (E) not is the same order. You have to find out the
exact code for the underlined letter in the world. The number of that code is the answer.
Please not that the same letter appearing in other word(s) may be coded differently.

22. ABOVE
(A) q (B) g (C) v (D) b (E) q

23. COVER
(A) u (B) y (C) q (D) g (E) f

24. BLAST
(A) i (B) e (C) q (D) p (E) d

ANSWERS

Que. 1 2 3 4 5 6 7 8 9 10 11 12
Ans. A B A A B B C C B D B D
Que. 13 14 15 16 17 18 19 20 21 22 23 24
Ans. C D B C A D A D D D E D

19
ALPHABET - TEST &
NUMBER RANKING

ALPHABETICAL ORDER :
You have to arrange these word in order in which they are arranged in a dictionary. In a dictionary
the words are placed in alphabetical order w.r.t the second alphabet of the word and so on (that
is, third alphabet, fourth alphabet.....)

Direction : Arrange in the correct alphabetical order.

Ex.1 Plane, Plain, Plan, Plenty, Player, prayer, Place.


Sol. The given words can be arranged in the alphabetical order as :
Place, Plain, Plan, Player, Plenty, Prayer.

Ex.2 Arrange the given words in alphabetical order and tick the one that comes last.
Heavy, Heredity, Hesitate, Hedge, Hero, Haste, History, Hindrance
Sol. The given words can be arranged in the alphabetical order as:
Haste, Heavy, Hedge, Heredity, Hero, Hesitate, Hindrance, History
Clearly, History comes last.

Ex.3 Arrange the given word in the order they occur in dictionary.
1. SIGN 2. SOLID 3. SCENE 4. SIMPLE
(A) 3, 1, 2, 4 (B) 3, 1, 4, 2 (C) 3, 4, 1, 2 (D) 3, 4, 2, 1
Sol. (B) The correct alphabetical order of the given words is :
SCENE, SIGN, SIMPLE, SOLID Thus, the correct sequence is 3, 1, 4, 2
Directions : (4) In the following questions, a group of letters is given which are numbered 1,2,3,4,5
and 6. Below are given four alternatives containing combinations of these numbers. Select
that combinations of numbers so that letters arranged accordingly, form a meaningful
word.
Ex.4 RRA OUH
123456
(A) 1, 3, 4, 5, 6, 2 (B) 2, 3, 6, 4, 5, 1 (C) 6, 3, 2, 4, 5, 1 (D) 3, 5, 2, 6, 4, 1
Sol. (D) The given letter, when arranged in the order 3, 5, 2, 6, 4, 1 Fro m the word AUTHOR.

20
Ex.5 In the word PARADISE how many pairs of letters are there which have as many letters between
them in the word as in the alphabet ?
(A) None (B) One (C) Two (D) Three
Sol. (D) Letter in the given word Letter in the alphabet series
(i) P A R PQR
(ii) A R A D ABCD
(iii) A D I S E A BDE

Ex.6 Number of letters skipped in between adjacent letters in the series decreases by two. Which of
the following series observes this rule ?
(A) EPVAF (B) GPWBE (C) UVJOP (D) XFMQU
Sol. (B) G H I J K L M N O P Q R S T U V W X Y Z A B C D E
8 6 4 2
Clearly, in letter series GPWBE, the number of letter skipped in between adjacent letters in the
series is decreases by two.

Ex.7 In the alphabet series which letter is midway between 22nd letter from the left and 21st letter from
the right ?
(A) L (B) M (C) O (D) None of these
Sol. (D) Consider the English alphabet :
22 Letters from the left
A B C D E F G H I J K L M N O P Q R S T U V W X Y Z
21 Letters from the right
By counting 21 letters from the first and 22 letters from the left, we get the following sequence in
which N comes exactly middle.
FGHIJKLM N OPQRSTUV

Ex.8 In the first half of the alphabet is written in the reverse order, which of the following will be the 19th
letter from your right ?
(A) H (B) F (D) (D) E
Sol. (B) The new alphabet series is :
MLKJIHGFEDCBNAPQRSTUVWXYZ
Nineteenth letter from the right will be letter F.

21
Ex.9 It is possible to make a meaningful word out of the second, the fourth, the fifth and the eighth
letters of the world ILLOGICAL then which of the following will be the third letter of the so formed
word ? If more than one word can be formed then give X and the answer.
(A) A (B) G (C) O (D) X
Sol. The second, fourth, fifth and eighth letters of the world ILLOICAL are, L, I, G, A respectively. The
words formed are GOAL and GOAL.

Ex.10 In the following scrambled letters are rearranged to from the name of a city, which letter will
appear in the middle /
AIDMURA
(A) M (B) R (C) U (D) D
Sol. (C) The city name is MADURAI and letter U exists exactly in the middle.

Ex.11 Find which one word can not be made from the letters of the given word.
CORRESPONDING
(A) DROPERS (B) SUPERIOR (C) GRINDER (D) DISCERN
Sol. (B) The word CORRESPONDING contains all the letters of the word SUPERIOR except U. So,
the word SUPERIOR can’t be formed.

NUMBER RANKING :

Ex.12 In the following number series, how may 8’s are there with are immediately preceded by a
number which does not divide it but followed by a number which divides it ?
28283 858853282384715838286
(A) 1 (B) 2 (C) 3 (D) 4
Sol. (C) As per the question
28283858853282384715838286
Thus, three such numbers are there

Ex.13 In a row of girls, Mardula is 18th from the right and Sanjana is 18th from the left. If both of them
exchange their position, Sanjana becomes 25th from the left, how many girls are there in the
row?
(A) 40 (B) 41 (C) 42 (D) 35
Sol. (C) Sanjana’s new position is 25th from left. But it is the some as Mrudula’s earlier position which
is 18th from the right.
then the total number of girls are = (rank from left + rank from right( - 1
= (18 + 25) - 1 = 43 - 1 = 42.

22
Ex.14 In a quene of boys Sohan is 9th from the back. Ramesh’s place is 8th from the front. Radhey is
standing in the middle of the two. What would be the minimum number of boys standing in the
queue ?
(A) 8 (B) 10 (C) 12 (D) 14
Ramesh’s position from the front

Sol. (B) Case - I 1 2 3 4 5 6 4 7 8 9

Sohan’ position from the back


In this case there is only even number of boys in between Ramesh and Sohan. So, Radhey
cannto stand in the middle of two.
Ramesh’s position from the front

Case- II 1 2 3 4 5 6 7 8 9 10

Sohan’s position from the back


In this case there is odd number of boys (i.e.,5) in between Ramesh and Sohan. So, Radhey can
stand in the exact middle of two. So, the minimum number of boys standing in queue are 10.

Ex.15 In the number from 1 to 45 which are exactly divisible by 3 are arranged is ascending order,
minimum number being on the top, which would come at the ninth place form the top ?
(A) 18 (B) 21 (C) 24 (D) 27
Sol. (D) The required numbers in ascending order are : 3, 6, 9, 12, 15, 18, 21, 24, 27 ,30, 33, 36, 39,
42, 45. The 9th number from the top is 27.
PRACTICE EXERCISE
1. Arrange the word in the alphabetical order and tick the one that comes second.
(A) Explosion (B) Emergency (C) Ecstasy (D) Eager
Directions : (2 to 3) In each of the following questions, a group of letter is given which are
numbered 1,2,3,4,5 and 6. Below are given four alternatives containing combinations of
these numbers. Select that combination of numbers so that letters arranged accordingly,
form a meaningful world.
2. K A T C EL
1 2 3 4 5 6
(A) 4, 2, 3, 1, 5, 6 (B) 1, 2, 4, 5, 6, 3 (C) 6, 5, 3, 2, 4, 1 (D) 3, 2, 4, 1, 6, 5
3. I N L A S G
1 2 3 4 5 6
(A) 6, 1, 3, 5, 4, 2 (B) 5, 1, 6, 2, 4, 3 (C) 3, 4, 6, 1, 2, 5 (D) 2, 4, 3, 6, 1, 5

23
4. How many such letters are there in the word CREATIVE, which have as many letters between
them in the word as in the alphabet ?
(A) 1 (B) 2 (C) 3 (D) 4

5. How many pairs of letter in the word BRIGHTER have as many letters between them in the word
as in the alphabet >
(A) 2 (B) 3 (C) 4 (D) more than 4

6. Number of letters skipped between adjacent letter in the series is in the order of 2, 5, 7, 10. Which
of the following series observes the rule given above ?
(A) CEGLT (B) FNKOT (C) QTZHS (D) SYBEP

7. If the alphabets are written in the reverse order, which letter will be the fifth letter to the right of the
fourteenth letter from the left.
(A) R (B) I (C) S (D) H

8. In the alphabet series the first and the second letters interchange their positions and similarly the
third and the fourth letter, the fifth and the sixth letters and so on, which letter will be the 17th from
your right ?
(A) H (B) I (C) F (D) K

9. If the first and the firth letters in the word ORDINARY are interchanged, the second and the sixth,
the third and the seventh and so on what will be the fifth letter from the right and after
rearrangement ?
(A) R (B) I (C) Y (D) N

10. In the first and second letters in the word COMMUNITAIONS were interchanged, also the third
and the fourth letters, the fifth and sixth letters and so on, which would be the tenth letter counting
from your right ?
(A) N (B) A (C) T (D) U

11. If it is possible to make a meaningful word with the first, the fifth, the sixth and the eleventh letters
of the word COURAGEOUSLY, which of the following will be third letter of that word ? If no such
word can be made, give X as the answer and if more than one such word can be made, give M as
the answer.
(A) O (B) A (C) G (D) X

12. If it is possible to from a word the first, fourth, seventh and eleventh letters of the word
SUPERFLUOUS, write the first letter of that word. Otherwise X is the answer.
(A) S (B) L (C) X (D) E

Directions : (13 to 14) In each of the following questions, find which one word can not be made
from the letters of the given word.

13. TEMPERAMENT
(A) METER (B) PETER (C) TENTER (D) TESTER

24
14. RAPPROCHEMENT
(A) REPRESENT (B) REPROACH (C) PHANTOM (D) CEMENT

15. How many odd numbers are there in the following sequence which are immediately followed by
an odd number ?
5147398526315863852243496
(A) 2 (B) 3 (C) 4 (D) More than 4

16. How many 6’s are there in the following sequence, which are either immediately preceded by 2 or
immediately followed by 9 ?
562436926716476 8263469862
(A) 1 (B) 2 (C) 3 (D) 4

17. Anil and Sunil are ranked seventh and eleventh respectively from the top in a class of 31
students. What will be their respective ranks from the bottom in the class ?
(A) 20th and 24th (B) 24th and 20th (C) 25th and 21st (D) 26th and 22nd

18. If all the numbers from 7 to 59, which are divisible by 3 are arranged in descending order then
which number will be at 10th place from the bottom ?
(A) 36 (B) 39 (C) 30 (D) 27

19. In a row of girls, Rina and Mona occupy the ninth place from the right end and tenth place from
the left end, respectively. If they interchanged their places, Rina and Mona occupy seventeenth
place from the right and eighteenth place from the left, respectively. How many girls are there in
the row ?
(A) 25 (B) 26 (C) 27 (D) Data inadequate

20. Three persons P, Q and R are standing in a queue. There are five persons between P and Q and
eight persons between Q and R. If there be three persons ahead of R and 18 persons behind P,
what could be the minimum number of persons in the queue ?
(A) 38 (B) 37 (C) 25 (D) 28

ANSWERS

Que. 1 2 3 4 5 6 7 8 9 10
Ans. C D B C B C A B C A
Que. 11 12 13 14 15 16 17 18 19 20
Ans. D B D A D D C A B C

25
MATHEMATICAL
OPERATIONS

You are provided with substitutes for various mathematical symbols. This is called Substitution
method. You are required to put in the real signs in the given equation and then solve the
questions.
Note :
While attempting to solve a mathematical expression, proceed according to the rule BODMAS -
that is, Brackets, Of, Division, Multiplication, Addition, Subtraction.

Ex.1 (48 - 12) ÷ 4 + 6 ÷ 2 × 3 = ?


Sol. (48 - 12) ÷ 4 + 6 ÷ 2 × 3 = 36 ÷ 4 + 6 ÷ 2 × 3 (Solving Bracket)
=9+3×3 (Solving Division)
=9+9 (Solving Multiplication)
= 18 (Solving Addition)

Ex.2 If x stands for addition, < for substraction, + stands for division, > for multiplication, - stands
for equal to, ÷ for greater than and = stands for less than state which of the following is true ?
(A) 3 × 2 < 4 ÷ 16 > 2 + 4 (B) 5 > 2 + 2 = 10 < 4 × 2
(C) 3 × 4 > 2 - 9 + 3 < 3 (D) 5 × 3 < 7 ÷ 8 + 4 ÷ 1
Sol. (B) Using proper nations, we have
(A) given statement is 3 + 2 - 4 > 16 × 2 ÷ 4 or 1 > 8, which is not true.
(B) given statement is 5 × 2 ÷ 2 < 10 - 4 + 2 or 5 < 8, which is true.
(C) given statement is 3 + 4 × 2 = 9 ÷ 3 - 3 or 11 = 0, which is not true.
(D) given statement is 5 + 3 - 7 > 8 ÷ 4 + 1 or 1 > 3, which is not true.

Ex.3 If + is xm - is +, x is ÷ and ÷ is -, then what is the value of given equation


21 ÷ 8 + 2 - 12 × 3 = ?
(A) 14 (B) 9 (C) 13.5 (D) 11
Sol. (B) Using the proper signs, we get
21 - 8 × 2 + 12 ÷ 3 = 21 - 8 × 2 + 4
= 21 - 16 + 4 = 9.

26
Ex.4 Find out to sign to be interchanged for making the questions correct
10 + 10 ÷ 1 - 10 × 10 = 10
(A) + and - (B) + and × (C) ÷ and x (D) + and ÷
Sol. (B) By making the interchanges given in (A), the equation as
10 - 10 ¯ 10 + 10 × 10 = 10 or 109 = 10 which is false
By making the interchanges given in (B), the equation as
10 × 10 ÷ 10 - 10 + 10 = 10 or 10 = 10 which is true
By making the interchanges given in (C), the equation as
10 + 10 × 10 - 10 ÷ 10 = 10 or 109 = 10 which is false
By making the interchanges given in (D), the equation as
10 ÷ 10 + 10 - 10 ÷ 10 = 10 or - 89 = 10 which is false

Direction : (5) In the following questions find the relationship that can definitely be deducted on
the basis two relationship given. The symbols used are as follows :
□ means greater then, ∆ means less than - means not equal to + means equal to

Ex.5 If 8A ∆ 6B and 3B ∆ 4C, therefore


(A) C □ A (B) C ∆ A (C) 2C + A (D) C □ 2A
Sol. (A) From the questions we get 8 A < 6 B or 4A < 3B and 3B M 4C which implies that 4A < 3B <
4C. From this relationship we conclude that 4a < 4C or A < C i.e., C □ A.

Ex.6 Which of the following conclusion is correct according to the given expression and symbols ?
A : >| B:> C :≠ D:=
E : <| F:<
Expression (pEq) and (qEr)
(A) pEr (B) pEr(C) rBp (D) rBp
Sol. (A) pEq and qEr ⇒ p <| q and q <| r ⇒ p <| r ⇒ p Er

Ex.7 If A + D > C + E, C + D = 2B nd B + E > C + D, it necessarily follows that


(A) A + B > 2D(B) B + D > C + E (C) A + D > B + E (D) A + D > B + C
Sol. (D) A + B > C + E
⇒ A + D > (2B - D) + E (∴ C + D = 2B)
⇒ A + D > (B + E) + (B - D)
⇒ A + D > (C + D) + (B - D)
⇒ A + D > B + C.
Direction : (8) In the questions given below, use the following notations :
A “ B means ‘add B to ‘;
A ‘ b means ‘subtract B from A’;
A @ B means ‘divide A by B’;
A , B means ‘multiply A by B’;
Now, answer the following question.

27
Ex.8 The time taken by two running trains in crossing each other is calculated by dividing the sum of
the lengths of two trains by the total speed of the two trains. If the length of the first train is L1, the
length of the second train is L2, the speed of the first train is V1 and the speed of the second train
is V2, which of the following expression would represent the time taken ?
(A) (L1 “ L2) , (V2” V2) (B) {L1 “ L2) @ (V1 “ V2)
(C) [(L1 “ L2) @ (V1 “ V2)] , 60 (D) (L1 ‘ L2) @ (V1’ V2)

sum of lenghts of two trains


Sol. (B) Clearly, time taken =
total speed of two trains
L1 + L 2
= = (L1 “ L2) @ (V1 “ V2)
V1 + V2

Directions : (9 to 10) The following symbols have been used.


x Stands for equal to
< Stands for not equal to
- Stands for greater than
+ Stands for not greater than
> Stands for less than
= Stands for not less than

Ex.9 If p + q = r, then it is not possible that


(A) p > q > r (B) p < q + r (C) p × q > r (D) p + q × r

Sol. (C) With the notations given,


p + q = r means p ≤ q ≥ r
From option (A), p > q > r means p < q ≠ r, this is true.
From option (B), p < q + r means p ≠ q ≤ r, this is true.
From option (C), p × q > r means p = q < r, this is not true.
From option (D), p + q × r means p ≤ q = r, this is true.

Ex.10 If p = q = r, then it is possible that ,


(A) p < q > r (B) p x q x r (C) p > q +r (D) p > q > r

Sol. (B) With the notations given,


p = q = r means p ≥ q ≥ r
From option (A), p < q < r means p ≠ q < r, this is not true.
From option (B), p × q × r means p = q = r, this is true
From option (C), p > q + r means p < q ≤ r, this is not true.
From option (D), p > q > r means p < q < r, this is not true.

28
Directions : (11 to 13) In the following questions the symbols $, @ ⊂, ⊃ and ≠ are used with the
following meaning.

A $ B means A is greater than B


A @ B means A is either greater than or equal to B
A ⊂ B means is A is equal to B
A ⊃ B means A is smaller than B
A ≠ B means A is either smaller than or equal to B
Now is each of the following questions assuming the given statements to be true, find
which of the two conclusion I and II given below them is / are definitely true ? Given
answer (A) if only conclusion I is true, (B) if only conclusion II is true (C) if neither I nor Ii is
true (D) if both I and II are true.

Ex.11 Statements : P @ Q, M ≠ N, N ⊂ Q
Conclusion : I P $ M
II. N ≠ P
Sol. (B) As per the statement
P ≥ Q = N ≤ M, Conclusion
I . P > Q.
II. N ≤ O
Only conclusion (II) is completely correct

Ex.12 Statements : D ≥ X, F @ Y, D $ F
Conclusion : I. X @ Y
II. Y ≠ D
Sol. (C) As per the statement
X = D > F ≥ Y, from the conclusions
I. X ≥ Y
II. Y ≤ D
No conclusion follows

Ex.13 Statements : M ⊂ P, S $ T, M @ T
Conclusions : I. T ≤ P
II. S ⊃ T
Sol. (A) As per the statement
P = M ≥ T < S, from the conclusions
I. T ≤ P
II. S < T
Conclusion (I) is correct

29
PRACTICE EXERCISE

Directions : (1) In each of the following questions, different alphabets stand for various symbols
as indicated below :
Addition : O Subtraction : M Multiplication : A
Division : Q Equal to : X Greater than : Y
Less than : Z
Out of the four alternatives given in these questions, only one is correct.
1. (A) 32 X 8 Q 2 A 3 Q 1 A 2 (B) 10 X 2 A 3 A 2 M 2 Q 1
(C) 2 Y 1 A 1 Q 1 O 1 A 1 (D) 16 Y 8 A 3 O 1 A 2 M 2

2. If ÷ means +, - means ÷, × means - and + means ×, then


(32 × 8) − 8 × 2
=?
4 + 18 × 8 + 9 ÷ 1
(A) 0 (B) 1 (C) 12 (D) None of these

3. If a means ‘plus’ b means ‘minus’, c means ‘multiplied by’ and d means ‘divided by’ then 16c 12 b
6d 2a 17 = ?
(A) 65 (B) 55 (C) 216 (D) 206

4. If > denote +, < denotes -, + denotes ÷, ∧ denotes ×, - denotes =, x denotes > and = denotes <,
choose the correct statement in the following questions.
(A) 14 > 18 + 9 = 16 + 4 > 1 (B) 4 > 3 ∧ 8 < 1 - 6 + 2 > 24
(C) 3 < 6 ∧ 4 > 25 = 8 + 4 > 1 (D) 12 > 9 ∧ 3 < 6 × 25 + 5 > 6

5. If = 7, = 27, = 81 then =?
(A) 690 (B) 689 (C) 780 (D) 789

6. Correct the following equation by interchanging two sign


5 - × 45 + 15 ÷ 3 = 5
(A) + and - (B) × and + (C) × and ÷ (D) × and -

Directions : (7 to 8) Answer the questions on the basis of the information given below, If ‘$’
represents ‘+’, ‘,’ represents ‘-’ ‘#’ represents ‘×’ and ‘@’ represent ‘/’ then answer the
following questions based on the above given representation.

7. What is the value of 4 # 3 $ 10 @ 5 $ 8 # 2, 18 ?


(A) 10 (B) 12 (C) 6.8 (D) 11.2

30
8. Which of the following has the value equivalent of 5$ 6 # 2 $ 8 @ 4 ?
(A) 4 # 7, 12 $ 2 # 1 (B) 8 # 2 , 3 $ 6 @ 3 (C) 8 @ 2 , 3 $ 6 # 3 (D) 4 $ 7 , 12 $ 2 # 1

Direction : (9) In the following questions find the relationship that can definitely be deducted on
the basis two relationship given. The symbols used are as follows :
□ means ‘greater than’, ∆ means ‘less than’, - means ‘not equal to’, + means ‘equal to’

9. IF B □ D, D ∆ C, C □ A and B + A, therefore
(A) C □ B (B) C ∆ B (C) C - B (D) Can’t be determined

Directions : (10) In the following questions given below, use the following notations :
A “ B means “add B to A”
A ‘ B means “subtract B from A”
A @ B means “divide A by B”
A , B means “multiply A by B”

10. The total airfare is calculated by adding 15% basic fare as fuel surcharge, 2% of basic fare as
IATA charges and Rs. 200 s airport tax to the basic fare. if the basic fare of a section is B, which
of the following will represent the total fare ?
(A) B ‘ (B , 15) @ 100” (B , 2) @ 100” 200
(B) B” (B , 15) @ 100” (B, 2) @ 100” 200
(C) B” (B , 15) @ 100” (B, 2) @ 100” 200
(D) B” (B, 15) @ 100” (B, 2) @ 100” 100

Direction : (11 to 12) The following symbols have been used


× stands for equal to
< stands for not equal to
- stands for greater than
+ stands for not greater than
> stands for less than
= stands for not less than

11. If p × q × r, then it is not possible that


(A) p + q = r (B) p = q + r (C) p + q + r (D) p = q = r

12. If p + q - r, then it is not possible that


(A) p × q = r (B) p + q < r (C) p = q = r (D) p - q - r

31
Directions : (13 to 14) In the following questions :
∆ means ‘is equal to’
□ means ‘is not equal to’
+ means ‘is greater than’
- means ‘is less than’
× means ‘is not greater than’
÷ means is not less than
Now select the correct alternative in each of the following questions :

13. a - b - c implies
(A) a - b + c (B) b + a - c (C) c × b + a (D) b + a ÷ c

14. a + b + c does not imply


(A) b - a + c (B) c - b - a (C) c - a + b (D) b - a - c

Directions : (15 to 19): In the following questions the symbol @, @ , =, © and @ are used with

following meaning :
P@Q → P is greater than Q
P @ Q → P is either greater or equal to Q

P @ Q → is smaller than Q

P @ Q → P is either smaller than or equal to Q

P = Q → P is equal to Q.
Now in each of the following equations, assuming the given statement, to be true, find
which of the two conclusions I and II given below them is/are definitely true.
Given answer (A) If only conclusion I is true, give answer (B) it only conclusion II is true,
given answer (C) if either I or Ii is true, given answer (D) if neither I nor II is true, give
answer (E) if both I and II are true.

15. Statement : B @ V, K © C, C @ B

Conclusions : I. V @ C
II. B @ K

16. Statement : K @ T, S = K, T R
Conclusions : I. S @ R
II. T = R

17. Statement : U = M, P @ U, M @ B

Conclusion : I. P = B
II. P @ B

32
18. Statement : L @ N, J P, IP @ L

Conclusions: I. J = L
II. P = N

20. Statement : H @ G, D @ E, H = E

Conclusions : I. D @ H
II. G © D

20. In the correctly worked out multiplication problem at the below, each letter represent a different
digit. What is the value of B ?
A A
X A B
BB

AA X
A 3B

(A) 1 (B) 2 (C) 4 (D) 5

ANSWERS

Que. 1 2 3 4 5 6 7 8 9 10
Ans. B B D B B D B C A C
Que. 11 12 13 14 15 16 17 18 19 20
Ans. A D B D B D C D E B

33
PUZZLE TEST

Direction : (1 to 5) Study the given information carefully and answer the questions that follow.
There are four people sitting in a row : one each from India, Japan, USA and Germany, but
not in that order,
I. They are wearing caps of different colours - green, yellow, red and white, not
necessarily in that order.
II. One is wearing a kurta and one a T-shirt
III. The India in wearing a green cap and a jacket.
IV. The American is not seated at either end.
V. The persons with kurta and T-shirt are sitting next to each other.
VI. The persons with kurta wears a red cap and sits next to the Japanese.
VII. The Japanese wears a shirt and is not seated at either end.
VIII. The man with white cap wears T-shirt and is seated at one end.

Ex.1 Who wears the T-shirt ?


(A) Indian (B) Japanese (C) American (D) German

Ex.2 Who is wearing a kurta ?


(A) Indian (B) Japanese (C) American (D) German

Ex.3 What is the colour of the cap worn by the Japanese ?


(A) Red (B) Green (C) Yellow (D) White

Ex.4 Who precedes the man wearing T-shirt ?


(A) Indian (B) Japanese (C) American (D) German

Ex.5 Who precedes the man wearing jacket ?


(A) Indian (B)German (C) Japanese (D) Cannot say

34
Sol. (1 to 5) : According to the question
From III, Indian is wearing a green cap and a Jacket ....(1)
From VI, Kurta is worn along with red cap and sits next to Japanese ....(2)
From VIII, T - Shirt with white cap combination is seated at one end ....(3)
So form (1) (2) (3), VII and I we conclude that the Japanese wear a shirt of yellow colour.
From IV, V, VI and VII, we conclude that the placement of people will be like
(i) (ii) (iii) (iv)
German American Japanese Indian
From (2) and IV, we arrive at the following table with the help of which rest of the questions can
be solved very easily.

Nationality German American Japanese Indian


Clothes T-shirt Kurta Shirt Jacket
Caps Whitecap Redcap Yellow Cap Green Cap

Sol.1 (D) German wears the T-shirt.

Sol.2 (C) American is wearing a kurta.

Sol.3 (C) Yellow is the colour of the cap worn by the Japanese.

Sol.4 (C) American precedes the man wearing T-shirt.

Sol.5 (C) Japanese precedes the man wearing jacket.

Directions : (6) Examine the following statements :


I. Either A and B are of the same age or A is older than B.
II. Either C and D are of the same age or D is older than C.
III. B is older than C.

Ex.6 Which one of the following conclusions can be drawn from the above statements ?
(A) A is older than B (B) B and D are of the same age
(C) D is older than C (D) A is older than C
Sol. (D) According to the given statements the following sequence are possible
Either (i) A = B > C = B or (ii) A > B > C, D > C

35
Directions : (7 to 8) Read the following information carefully and answer the questions given
below it.
I. Seven books are placed one above the other in a particular way.
II. The history book is placed directly above the civics book.
III. The geography book is fourth from the bottom and the English book is fifth from the
top.
IV. There are two books in between the civics and economic books.

Ex.7 To find the number of books between the civic and the science books, which other extra piece of
information is required, from the following ?
(A) There are two books between the geography and the science books.
(B) There are two books between the mathematics and the geography books.
(C) There is one book between the English and the science books.
(D) The civics book is placed before two books above the economic book.

Ex.8 To know which three books are kept above the English book, which of the following additional
pieces of information, if any, is required ?
(A) The economics book is between the English and the science books.
(B) There are two books between the English and the history books.
(C) The geography book is above the English book.
(D) No other information is required.
Sol. (7 to 8) : According to the given question from II, we get
History .....(1) From III, we get
Civics 

_______ 
_______ 
______ 
Geography  ....(2) From IV (1) and (2), we get
English 
_______ 
_______ 
_______ 

_______ 
History 
Civics 

Geography 
English 
Economics 
_______ 

Since history and civics cannot be at any other place than this, according to the given conditions.
On the basis of this very arrangement, rest of the questions can be solved very easily.
7. (C) Clearly, C gives us the clue that the science book is placed at the bottom. Thus, we know that
there are three books between the civics and science books.

8. (D) Clearly, history, civics and geography are the three books kept above the English book. To
deduce this, no additional information is required.

36
Directions : (9 to 10) A five -member team that includes Rama, Shamma, Henna, Reena, and Tina,
is planning to go to a science fair but each of them put up certain conditions for going.
They are as follows :
I. If Rama goes, then at least one amongst Shamma and Henna must go.
II. If Shmma goes, then Reena will not go.
III. If Henna will go, then Tina must go.
IV. If Reena goes, then - Henna must go.
V. If Tina goes, then Rama must go but Shamma cannot go.
VI. If Reena plans not to go fair, then Rama will also not go.
Ex.9 If it sure that Henna will go to the fair, then who among the following will definitely go ?
(A) Rama (B) Shamma (C) Reena (D) Rama and Reena
Sol. (D) It is clear using conditions (I) and (IV). That Rama and Reena will go to the fair.

Ex.10 If Tina does not go to the fair, which of the following statements must be true ?
(i) Henna cannot go
(ii) Shamma cannot go
(iii) Reena cannot go
(iv) Rama cannot go
(A) (i) and (ii) (B) (iii) and (iv) (C) (i), (iii) and (iv) (D) (i) and (iv)
Sol. (C) Using condition III, (i) is true.
Thus, using condition IV, (iii) is true.
And using conditions VI, (iv) is also true.
We cannot say anything about Shamma.

Directions : (11 to 12) Read the given information carefully and asnwer the questions that follow :
Ratan, Anil, Pinku and Gaurav are brothers of Rakhi, Sangeeta, Pooja and Saroj, not
necessarily in that order. Each boy has one sister and the names of bothers and sisters do
not begin with the smae letter. Pinku and Gaurav are not Saroj’s or Sangeeta’s brothers.
Saroj is not Ratan’s sister.

Ex.11 Pooja’s brother is


(A) Ratan (B) Anil (C) Pinku (D) Gaurav

Ex.12 Which of the following are brother and sister ?


(A) Ratan and Pooja (B) Anil and Saroj (C) Pinku and Sangeeta (D) Gaurav and Rakhi

37
Sol.(11 to 12) : As given that the names of brothers and sisters do not begin with the same letter and
Pinku and Gaurav and not Saroj or Sangeeta’s brothers, Pinku cannot be the brother of Pooja and
Hence he is the brother of Rakhi.
Now we have that Gaurav cannot be the brother of Saroj, Sangeta or Rakhi. Therefore Gaurav is
the brother of Pooja. As given that Saroj is not Ratan’s sister and Rakhi and Pooja can also not be
the sister’s of Ratan (From above conclusions), Ratan is the brother of Sangeeta. Anil will have to
be the brother of saroj as this is the only valid combination left. Therefore, we have this table
finally.

Brother Sister
Pinku Rakhi
Gaurav Pooja
Ratan Sangeeta
Anil Saroj

Sol.11 (D) Gaurav is Pooja’s brother

Sol.12 (B) Anil and Saroj are brother and sister.

Direction : (13) The ages of Mandar, Shivku, Pawan and Chandra are 32, 21, 35 and 29 years, not
in order Whenever asked they lie of their own age but tell the truth about others.
(i) Pawan says, “My age is 32 and Manda’s age is not 35”
(ii) Shivku says, “My age is not 209 and Pawan’s age is not 21”
(iii) Mandar says, “My age is 32.”

Ex.13 What is Chandra’s age ?


(A) 32 years (B) 35 years (C) 29 years (D) 21 years
Sol. (A) From the first statement, it is clear that Pawan’s age is not 32 years and Mandar’s age in not
35 years. From the second statement, it is clear that shivku’s age is 29 years and Pawan’s age is
not 21 years. Thus, from these two statements we get Pawan’s age as is 35 years. Now from the
third statement, Mandar’s age is not 32 years. thus, Mandar’s age is 21 years. Hence, we get
Chandra’s age as 32 years.

38
PRACTICE EXERCISE

Directions : (1 to 5) Read the following information carefully and answer the questions that follow.
I. There are six students (A, B, C, D, E and F) in a group. Each student can opt for only
three choices out of the six which are music, reading, painting, badminton, cricket and
tennis.
II. A, C and F like reading.
III. D does not lie badminton, but likes music.
IV. Both B and E like painting and music.
V. A and D do not like painting, but they like cricket.
VI. All students except one like badminton.
VII. Two students like tannins.
VIII. F does not like cricket, music and tennis.

1. Which pair of students has the same combination of choices ?


(A) A and C (B) C and D (C) B and E (D) D and F

2. Who among the following students like both tennis and cricket ?
(A) A and B (B) C (C) B and D (D) D

3. How many students like painting and badminton ?


(A) 1 (B) 2 (C) 3 (D) 4

4. Who among the following do not like music ?


(A) A, C and D (B) A, B and C (C) A, C and F (D) B, D and F

5. Which of the following is the most popular choice ?


(A) Tennis (B) Badminton (C) Reading (D) Painting

6. R earns more than H but not as much as T, M earns more than R. Who earns least among them ?
(A) R (B) T (C) H (D) M

7. Harish is taller than Manish but shorted than Suresh. Manish is shorter than Anil but taller than
Raghu. Who among them is the shortest having regard to height ?
(A) Anil (B) Manish (C) Raghu (D) Cannot be determined

39
Direction : (8 to 11) Read the following paragraph carefully and choose the correct alternative.
The office staff of XYZ corporation presently consist of three females A,B,C and five males
D,E F, G, H. The management is planning to open a new office in another city using three
males and two females of the present staff. To do so they plan to separate certain
individual who do not function well together. The following guidelines were established
1. Females A and C are not to be together
2. C and E should be separated
3. D and G should be separated
4. D and F should not be part of a team.

8. If A is chosen to be moved, which of the following cannot be a team ?


(A) ABDEH (B) ABDGH (C) ABEFH (D) ABEGH

9. If C and F are to be moved to the new office, how many combinations are possible ?
(A) 1 (B) 2 (C) 3 (D) 4

10. If C is chosen to the new office, which number of the staff cannot e chosen to go with C ?
(A) B (B) D (C) F (D) G

11. Under the guidelines, which of the following must be chosen to go to the new office >
(A) B (B) D (C) E (D) G

12. If D goes to the new office, which of the following is/ar true ?
I. C cannot be chosen
II. A cannot be chosen
III. H must be chosen
(A) I only (B) II only (C) I and II only (D) I and III only

Direction : (13 to 17 )
(i) There is a group of six persons P, Q, R, S, T and U from a family. They are
Psychologist, Manager, Lawyer, Jeweler, Doctor and Engineer.
(ii) The Doctor is grandfather of U, who is a Psychologist.
(iii) The Manager S is married to P.
(iv) R, the Jeweler is married to the Lawyer.
(v) Q is the mother of U and T.
(vi) There are two married couples in the family.

13. What is the profession of T ?


(A) Doctor (B) Jeweller (C) Manager (D) None of these

14. How is P related to T ?


(A) Brother (B) Uncle (C) Father (D) Grandfather

15. How many male members are their in the family ?


(A) One (B) Three (C) Four (D) Data inadequate

40
16. What is the profession of P ?
(A) Doctor (B) Lawyer (C) Jeweller (D) Manager

17. Which of the following is one of the pairs of couples in the family ?
(A) PQ (B) PR (C) PS (D) Cannot be determined

Directions : (18 to 19) Answer the questions on the basis of the information given below. 5 friends
Nitin, Reema, Jai, Deepti and Ashutosh are playing a game of crossing the roads. In the
beginning, Nitin, Reema and Ashutosh are on the one side of the road and Deepti and jai
are on the other side. At the end of the game, it wad found the Reema and Deepti are on
the one side and Nitin, Jai and Ashutosh are on the other side of the road. Rules of the
game are as follows :
I. One “Movement” means only one persons crosses the road from any side to the other
side.
II. Not two persons can cross the road simultaneously from any side to the other side.
III. Two persons from the same side of the roads cannot move in consecutive
“movements”.
IV. If one person crosses the road in a particular movement, he or she cannot immediately
move back to the other side.
V. Jai and Reema did not take part in first 3 movements.

18. What is the minimum possible number of movements that took place in the entire game ?
(A) 3 (B) 4 (C) 5 (D) 6

19. If number of movements are minimized in the game, then which of the following combination of
friends can never be together on one particular side of the road during the course of the game ?
(A) Nitin, Reema and Deepti (B) Nitin, Jai and Deepti
(C) Deepti, Jai and Ashutosh (D) Ashutosh, Nitin and Deepti

ANSWERS

Que. 1 2 3 4 5 6 7 8 9 10
Ans. C D C C B C C B A B
Que. 11 12 13 14 15 16 17 18 19
Ans. A A D D D A C A D

41
SEATING ARRANGEMENT

Directions : (1 to 5) Study the given information and answer the questions that following :
(i) P,Q,R,S,T,U and V are sitting is a row facing East.
(ii) R is on the immediate right of S.
(iii) Q is at an extreme end and has T as his neighbor.
(iv) V is between T and U.
(v) S is sitting third from the south end.

Ex.1 Who is sitting to the right of T ?


(A) P (B) C (C) C (D) U

Ex.2 Which of the following pairs of people are sitting at the extreme ends ?
(A) PQ (B) PS (C) QR (D) UB

Ex.3 Name the persons who is at the third place from the north end.
(A) T (B) U (C) V (D) S

Ex.4 Immediately between which of the following pairs of people is S sitting >
(A) PR (B) PU (C) RT (D) RU

Ex.5 Which of the conditions (i) to (iii) given above is not required to find out the place in which P is
sitting ?
(A) I (B) ii (C) iii (D) All are required
Sol. (1 to 5)
From the above information the sitting arrangement is as shown
1. (B) Clearly, V is sitting to the right of T.
2. (A) Clearly P and Q are sitting at the extreme ends.
3. (C) Clearly V is at the third place from the north ends.
4. (D) Clearly S is immediately between R and U.
5. (D) Clearly, all the conditions (i) to (v) given above is required to find out the place in which P
is sitting.

42
Direction : (6 to 8) : Six Persons P,Q,R,S,T and U are sitting in a circle facing one another front to
front. P is sitting gin front of Q, Q is sitting to the right of T and left of R, P is to the left U
and right of S.

Ex.6 Who is sitting opposite to R ?


(A) P (B) Q (C) S (D) U

Ex.7 Who is sitting opposite to S ?


(A) U (B) T (C) R (D) Q

Ex.8 Who is sitting between P are R ?


(A) S (B) T (C) U (D) Q
Sol. (6 to 8): Clearly, the circular arrangement is an shown

6. (D) Clearly U is opposite to R

7. (B) Clearly T is opposite to S

8. (A) Clearly S is sitting between P are R

Direction : (9 to 11) Read the following information and answer the questions given below it. Five
girls are standing in a circle facing the centre. Suman is between Lata and Asha. Mata is to
the right of Lata.

Ex.9 Who is the left of Asha if Rajani is the fifth girl ?


(A) Mamta (B) Suman (C) Lata (D) Rajani

Ex.10 If Suman and Mamta interchange their positions, who will be fourth to the left of Rajani ?
(A) Lata (B) Suman (C) Asha (D) Mamta

Ex.11 If Rajani and Asha interchange their position, then which of the following statements will be the
correct one ?
(A) Suman would be third to the left of Mamta (B) Asha would be between Lata and Rajani
(C) Lata would be second to the right of Asha (D) None of these

Sol. (9 to 11) :

43
9. (D) Clearly, Rajani is the left of Asha if Rajani is the fifth girl.

10. (C) Clearly, If Suman and Mamta interchange their positions, Asha will be fourth to the left of
Rajani.
11. (D) Clearly, If rajani and Asha interchange their positions, than alternatives (A), (B), (C) will not be
correct.

Direction : (12) Six friends are sitting around a circular table at equal instances from each other.
Ramola is sitting two places right of Komolika who is exactly opposite to Anu. Anu is
sitting on the immediate left of Pallavi, who is exactly opposite to Mandira, Natasha is also
sitting gat the table.

Ex.12 Which of the following statements is not correct?


(A) Natasha and Ramola are exactly apposite to each other.
(B) Mandira and Natasha are at equal distance from Komolika.
(C) Angle subtended by Mandira and Natasha is same as the angle subtended by Ramola and
Pallavi at the centre of the table.
(D) Natasha is on the immediate left of Pallavi.
Sol. (D) On the basis of the analysis of the given information, Natasha is on the immediate left of
Pallavi.

PRACTICE EXERCISE

Directions : (1 to 5) Read the following information carefully and answer the question given
below:
(i) Six flats on a floor in two rows facing north and south are allotted to P,Q,R,S,T and U.
(ii) Q gets a north facing flat and is not next to S.
(iii) S and U get diagonally opposite flats.
(iv) R next to U, gets a south facing flat and T gets a north facing flat.

1. Whose flat is between Q and S ?


(A) T (B) U (C) R (D) P

2. The flats of which of the pairs other than SU, is diagonally opposite to each other ?
(A) PT (B) QP (C) QR (D) TS

3. In the flats of T and P are interchanged, whose flat will be next to that of U ?
(A) Q (B) T (C) P (D) R

4. Which of the combinations get south facing flats ?


(A) URP (B) UPT (C) QTS (D) data inadequate

44
5. To arrive at the answers to the above questions, which of the following statements can be
dispensed with ?
(A) None (B) Only (i) (C) only (ii) (D) (iii) only

Directions : (6 to 8) Study the given information carefully and answer the questions that follow :
In a swimming race, five participants - A, B, C, D and E take part. Lane 1 is extreme left and
lane 5 is extreme right. The following conditions exists.
I. B and E not swimming adjacent to each other.
II. D is not in one of the extreme (outermost) lanes.
III. A is to the left of C.

6. If B in lane 3, A in lane 1, then C could be in


(A) lane 4
(B) lane 2
(C) lane 2 or 4
(D) Situation violates the conditions

7. If B is lane 4 and C is lane, then E could be in


(A) lane 1 (B) lane 2
(C) lane 1 or 2 (D) Situation violates the conditions.

8. If D is to the left of A, then D can be in


(A) lane 2 (B) lane 3 only (C) lane 2 or 3 (D) None of these

Direction : (9 to 11) : A, B, C and D are to be seated in a row. But C and D cannot be together. Also
B cannot be at the third place.

9. Which of the following must be false ?


(A) A is a the first place (B) A is at the second place
(C) A is at third place (D) A is at the fourth place

10. If A is not at the third place, then C has which of the following option ?
(A) The first place only (B) The third place only
(C) The first and third place only (D) Any of the places

11. If A and B are together, then which of the following must be necessarily true ?
(A) C is not at the first place (B) A is the third place
(C) D is at the first place (D) C is at the first place

Direction : (12) Refer to the data below and answer the questions that follows :
There are nine chairs in a row, each numbered 1 to 9 from left to right. Six friends are
sitting on these chairs Megha, Sapna and Riya are neither sitting at chair 1 nor at chair
numbered 9. Beena and megha does not have anybody sitting adjacent to them. There is
only one empty chair between Megha and Riya. Charu is adjacent to both Jiya and Riya.
Sapna is sitting at the seat numbered 2.

12. Megha is sitting on which of the following chairs ?


(A) 4 (B) 5 (C) 7 (D) 8

45
Directions : (13) Six friends Anil, Shehul, Rajesh, Kiran, Milind and Vinay are sitting around a
circular table. Following information about their seating arrangement is given.
I. Rajesh is to the immediate left of Kiran.
II. Neither Milind nor Kiran is the immediate neighbor of Shehul.
III. Anil is sitting between Milind and Shehul.
IV. Anil is not sitting exactly opposite to Rajesh.

13. Which of the following seating arrangements is definitely true according to the above information
(take in anticlockwise direction)
(A) Vinay, Shehul, Anil, Milind, Rajesh, Kiran (B) Vinay, Anil, Milind, Shehul, Rajesh, Kiran.
(C) Vinay, Milind, Anil, Shehul, Rajesh, Kiran (D) None of these

Directions : (14 to 18) Read the following information carefully and answer the questions given
below it.
I. Eight persons E,F,G,H,I,J,K and L are seated around a square table - two on each side.
II. There are three lady members and they are not seated next to each other.
III. J is between L and F.
IV. G is between I and F.
V. H, a lady member, is second to the left of J.
VI. L, a male member, is seated opposite of E, a lady member
VII. There is a lady member between F and I.

14. Who among the following is seated between E and H ?


(A) F (B) I (C) J (D) None of these

15. How many persons are seated between K and F ?


(A) One (B) Two (C) Three (D) Cannot be determined

16. Who among the following are the three lady members ?
(A) E, G and J (B) E, H and G (C) G, H and J (D) Cannot be determined

17. Who among the following is to the immediate left of F ?


(A) G (B) I (C) J (D) Cannot be determined

18. Which of the following is true about J ?


(A) J is a male member (B) J is a female member
(C) Sex of J cannot be determined (D) Position of J cannot be determined

Direction : (19 to 23) Study the following information to answer the given questions.

46
(i) Eight friends A, B, C, D, E, F, G and H are seated is a circle facing centre.
(ii) D is between B and G and F is between A and H.
(iii) E is second to the right of A.

19. Which of the following is A’s position ?


(A) left of F (B) Right of F (C) Between E and F (D) can’t be determined

20. Which of the following is C’s position ?


(A) Between E and A (B) Between G and E
(C) Second to the left of B (D) Can’t be determined

21. Who are the neighbors of D ?


(A) B and C (B) C and E (C) B and G (D) B and G or B and H

22. If the positions of B and G and D and A are interchanged then who is sitting between B and G in
new position.
(A) D (B) A (C) H (D) E

23. If B sitting opposite to C and H is sitting opposite to E then find who is sitting opposite to F ?
(A) B (B) G (C) A (D) D

ANSWERS

Que. 1 2 3 4 5 6 7 8 9 10 11 12
Ans. A B D A A C D A A C B C
Que. 13 14 15 16 17 18 19 20 21 22 23
Ans. D D C B C A B A C B B

47
BLOOD RELATIONS

Problems on Blood Relations involve analysis of information showing blood relationship among
members of a family. In the question, a chain of relationship is given in the form of information and
on the basis of this information relation between any two members of the chain in asked. Students
are supposed to be familiar with the knowledge of different relationship in a family.

Grandfather’s son Father or uncle


Grandmother’s son Father or uncle
Grandfather’s only son Father
Grandfather’s only son Father
Mother’s of Father’s mother Grandmother
Mother’s or Father’ father Grandfather
Grandfather’s only daughter –in – law Mother
Grandmother’s only daughter in law Mother
Mother’s or Father’s son Brother
Mother’s or Father’s daughter Sister
Mother’s or Father’s brother Uncle
Mother’s or Father sister Aunt
Husband’s or wife’s sister Sister-in-law
Husband’s or wife’ brother Brother-in-law
Son’s wife Daughter-in-law
Daughter’s husband Son-in-law
Brother’s son Nephew
Brother’s daughter Niece
Uncle or Aunt’s son or daughter Cousin
Sister’s husband Brother-in-law
Brother’s wife Sister-in-law

Ex.1 if P $ Q means P is the father of Q, P # Q means P is mother of Q, & P * Q means P is the sister
of Q. Then how is Q related to N if N # L $ P * Q
(A) grandson (B) granddaughter (C) nephew (D) data inadequate

Sol. (D) The sex of Q is not given hence the exact relation ship b/w N & Q cannot be established.

48
Ex.2 A is the brother of B,C is the brother of A. To establish a relationship between B & C, Which of the
following information is required.
I Sex of C II Sex of B
(A) only I is required (B) only II is required
(C) both I and Ii are required (D) Neither required
Sol. (B) It is clear that C is the Brother of B but how B is related to C depends on the sex of B.

Ex.3 Pointing towards a man in the photograph, lady said “the father of his brother is the only son of my
mother”. How is the man related to lady ?
(A) Brother (B) Son (C) Cousin (D) Nephew
Sol. (D) The father of this brother means “his father “ is the only son of my mother means” my brother”.
It means lady is the father’s sister of the man’s father.

Directions ; (4 to 7)
A + B means ‘A is father of B’
A-B means ‘A is wife of B’
A × B means ‘A is brother of B’
A ÷ B means ‘A is daughter of B’

Ex.4 P ÷ R + S + Q, which of the following is true ?


(A) P is daughter of Q (B) B is aunt of P (C) P is aunt of Q (D) P is mother of Q
Sol. (C) ‘S + Q’ & ‘R + S’ means R is the grandfather of Q. Now P ÷ R means P is daughter of R. This
clearly means P is aunt of Q

Ex.5 If P - R + Q, which of the following is true


(A) P is mother of Q (B) Q is daughter of P (C) P i aunt of Q (D) P is sister of Q
Sol. (A) P - R + Q, represents R is the father of Q, and P is the wife of R, ∴ P is the mother of Q

Ex.6 P × R ÷ Q, which of the following is true ?


(A) P is uncle of Q (B) P is father of Q (C) P is brother of Q (D) P is son of Q
Sol. (D) R is the daughter of Q & P is brother of R, ∴ P is son of Q

Ex.7 If P × R - Q which of the following is true.


(A) P is brother in law of Q (B) P is brother of Q
(C) P is uncle of Q (D) P is father of Q
Sol. (A) Clearly, P is related as brother in law to Q.

49
Ex.8 Soni, who is Dubey’s daughter, says to Preeti, “Your mother Shyama is the youngest sister of my
father, Dubey’s Father’s Third child is Prabhat”. How is Prabhat related to Preeti ?
(A) Uncle (B) Father (C) grandmother (D) Father is law
Sol. (A) Preeti’s mother shyama is youngest sister of Dubey & sister of Prabhat. Therefore Prabhat is
Preeti’s uncle.

Ex.9 Pointing toward a man in the photograph, Archana said, “he is the son of the only son of my
grandmother “. How is man related to Archana ?
(A) Cousin (B) Nephew (C) Brother (D) Son
Sol. (C) Only son of Archana’s grandfather means Archana’s father & his son is Archana’s brother.

Ex.10 Pointing towards a woman in the photograph, Rajesh said “the only daughter of her grandfather
(Paternal) in my wife “. How is Rajesh related to that woman
(A) uncle (Fufa) (B) Father (C) Maternal uncle (D) Brother
Sol. (A) Rajesh is the husband of woman’s father’s sister.

PRACTICE EXERCISE

1. Aaskah said to Mohit, “That boy in blue shirt is younger of the two brothers of the daughter of my
father’s wife”. How is the boy in blue shirt related to Aakash”.
(A) Father (B) Uncle (C) Brother (D) Nephew
2. Pointing to a person, Rohit said to Neha, “his mother is the only daughter of your father. “How is
neha related to that person ?
(A) Aunt (B) Mother (C) Daughter (D) Wife

3. ‘P + Q’ means ‘P is the brother of Q’, ‘P - Q means P is the mother of Q and ‘P × Q’ means ‘P is


the sister of Q’/ Which of the following means that M is the material uncle of R ?
(A) M - R + K (B) M + K - R (C) M + K × Q (D) None of these

4. ‘A + B’ means ‘A is the son of B’, ‘A - B’ means ‘A is the wife of B’. ‘A × B’ means ‘A is the brother
of B’, ‘A ÷ B’ means ‘A is the mother of B’, ‘A = B’ means ‘A is the sister of B’. Which of the
following represents P is the material - uncle of Q ?
(A) R × P ÷ Q (B) P × R ÷ Q (C) P + R ÷ Q (D) P + R × Q

50
5. Amit said, “This girls is the wife of the grandson of my mother.” How is Amit related to the girl ?
(A) Father (B) Father-in-law (C) Grandfather (D) Husband

6. Neelam, who is Rohit’s daughter, says to Indu, “Your mother Reeta is the younger sister of my
father, who is the third child of Sohanji.” How is Sohanji related to Indu ?
(A) Maternal - uncle (B) Grandfather (C) Father (D) Father-in-law

7. Pointing to a girl in the photograph, ? Ramesh said “Her mother’s brother is the only son of my
mother’s father.” How is the girl’s mother related to Ramesh ?
(A) Mother (B) Sister (C) Aunt (D) Grandmother

8. Pointing to a man in a photograph, Anita said “His brother’s father is the only son of my
grandfather” How is the Anita related to the man in the photograph ?
(A) Mother (B) Aunt (C) Sister (D) Daughter

9. Pointing to his son’s portrait, a man said to a woman, “His mother is the only daughter of your
mother”. How was the woman related to the man ?
(A) Sister (B) Mother (C) Wife (D) Daughter

10. Introducing a man, a woman said, “His wife is the only daughter of my father “. How that man was
related to the woman ?
(A) Brother (B) Father-in-law (C) Maternal Uncle (D) Husband

11. If Anil is the brother of the son of Sunil’s son, what is the relationship between Anil and Sunil ?
(A) Cousin (B) Brother (C) Nephew (D) Grandson

12. Pointing to a person, a man said to a woman, “His mother is the only daughter of your father”.
How was the woman related to the person ?
(A) Sister (B) Mother (C) Wife (D) Daughter

Directions : (13 to 15)P,Q,R,S,T,U,V & W are the family member. Q is the sister of V and V is the
brother of R.T. is the wife of P, whose father is W.S. is the husband of Q and U is the son of
V.P. is the father of Q.

13. How U is related with T ?


(A) Son (B) Mother (C) Grandson (D) Nephew

51
14. How S is related with R ?
(A) Son (B) uncle (C) Brother-in-law (D) Brother

15. How W is related with R ?


(A) Grand father (B) uncle (C) Son (D) Brother

Directions : (16 to 18) A,B,C,D,E & F are related to each other as given here. B is F’s daughter-in-
law. D is A’s only grand child. C is D’s only uncle. A has two children F and C, one male &
one female (not necessarily in the same order). E is the Father of C.

16. Who is the grand mother of D ?


(A) B (B) A (C) C (D) D

17. Who is the mother-in-law of B ?


(A) C (B) D (C) E (D) F

18. If a girls G is marred into the family, what is the relationship between G and D ?
(A) Mother (B) Aunt (C) Mother-in-law (D) Grand mother

Directions : (19 to 22) Read the following information carefully and answer the questions given
below :
There are six children playing football namely A, B, C, D, E and F. A and E are brothers. F
is the sister of E. C is the only son of A’s uncle. B and D are the daughter of fhe brother of
C’s father.

19. How is C related to F ?


(A) Cousin (B) Brother (C) Son (D) Uncle

20. How many male players are there ?


(A) One (B) Three (C) Five (D) Six

21. How many female players are there ?


(A) Two (B) Three (C) Five (D) one

22. How is D related to A ?


(A) Uncle (B) Sister (C) Niece (D) Cousin

52
Directions : (23 to 27) Study the information given below and answer the questions that follow :
There is a family of six persons A, B, C, D, E and F. They are Lawyer, Doctor, Teacher
Salesman, Engineer and Accountant. There are two married couples in the family. D, the
Salesman is married to the Lady Teacher. The Doctor is married to the Lawyer. F, the
Accountant is the son of B and brother of E. C, the Lawyer is the daughter-in-law of A. E is
the unmarried Engineer. A is the grandmother of F.

23. How is E related to F ?


(A) Brother (B) Sister (C) Cousin (D) Cannot be determined

24. What is the profession of B ?


(A) Teacher (B) Doctor (C) Lawyer (D) Cannot be determined

25. What is the profession of A ?


(A) Lawyer (B) Teacher (C) Doctor (D) Cannot be determined

26. Which of the following is one of the couples ?


(A) F and D (B) D and B (C) E and A (D) None of these

27. How is D related to F ?


(A) Grandfather (B) Father (C) Uncle (D) Brother

ANSWERS

Que. 1 2 3 4 5 6 7 8 9 10 11
Ans. C B B B B B A C C D D
Que. 12 13 14 15 16 17 18 19 20 21 22
Ans. B C C A B D B A B B D
Que. 23 24 25 26 27
Ans. D B B D A

53
DIRECTION SENSE TEST

There are four directions such as North, South, East and West. The word NEWS came from
North, East, West and South. There are four regions : North-East (i); North-West (ii) ; South-East
(iii); South-West (iv)

The directions OP, OS, OQ, and OR are :


North-East direction ; North-West direction ; South-West direction ; and South-East direction
respectively.

NOTE : The candidate must distinguish between the regions and directions, i.e., between North-East
regions and North-East direction. If you move with your face Eastwards, your left hand is
towards North and your right hand is towards South. Similarly the positions of the directions of
the hand can be fixed when you move in any of the other three directions.

Ex.1 Starting from a point ‘S’, Mahesh walked 25 metres towards South. he turned to his lef and
walked 50 metres. He then again turned to his left and walked 25 metres, He again turned to left
and walked 60 metres and reached a point ‘T’. How far is Mahesh from the point ‘S’ and in which
direction?
(A) 1 metres West (B) 25 metres North (C) 10 metres East (D) 25 metres West
Sol. (A) Mahesh is at a distance of 10 metres away and in West direction from his starting point S.

54
Ex.2 Village Cimur is 20 km, to the North of village Rewa. Village Rahate is 18 km to the East of village
Rewa. Village Angne is 12 km to the West of Chimur. If Sanay starts from village Rahate and
goes to village Angne, in which direction is he from his starting point ?
(A) North (B) North-West (C) South (D) South-East
Sol. (B) From the figure it is clear that A and B denote the starting and finishing points respectively. B
is to the North-West of Point A.

Ex.3 Ravi traveled 4 km straight towards south. He turned left and traveled 6 km straight, then turned
right and traveled 4 km straight. How far is he from the starting point ?
(A) 8 km (B) 10 km (C) 12 km (D) 18 km
Sol. (B) B is the finishing point and is 10 km. from the point A. The Aerial distance of A from B is 1 km,

calculated as below (AB)2 = (AD)2 + (DB)2 = (8)2 + (6)2 = 64 + 36 = 100


∴ AB = 10 km.

Ex.4 A man is facing North-West. he turns 900 in the clockwise direction , then 1800 in the

anticlockwise direction and then another 900 in the same direction. Which direction is he facing
now ?
(A) South (B) South - West
(C) West (D) South-East

Sol. (D) As shown in Fig. the man initially faces in the

direction OA. On moving 900 clockwise, he faces in the

direction OB. On further moving 1800 anticlockwise, he

faces in the direction OC. Finally on moving 900 anti-


clockwise, he faces in the direction OD, whish is South-
East.

55
Ex.5 Kishen walks 10 km towards North. Form there, he walks 6 km towards South. Then, he walk 3
km towards East. How far and in which direction is he with reference to his starting point ?
(A) 5 km, North (B) 5 km, North - East (C) 7 km - East (D) 7 km, West
Sol. (B) The movements of Kishen are as shown in Fig. (A to B, B to C and C to D).
AC = (AB - BC) = (10 - 6) km = 4 km. clearly, D is to the North-East of A.
∴ Kishen’s distance from starting point

A = AD = AC2 + C2 = 42 + 33 = 26 = 5 km.

So, Kishen is 5 km to the North-East of his starting point.

Ex.6 I am facing south. I turn 900 in the anti-clockwise direction and walk 30 m and then turning north I
walk 40 m and then turning west I go 60 m. Then turning left I walk 80 m. How far am I from the
starting point ?
(A) 30 m (B) 40 m (C) 50 m (D) 210 m
Sol. (C) According to the statement
Hence, the answer is 50 m

Ex.7 I am facing South. I turn right and walk 20 m. Then I turn right again and walk 10 m. Then I turn
left and walk 10 m and then turning right walk 20 m. Then I turn right again and walk 60 m. In
which direction am I form the starting point ?
(A) North (B) Northwest (C) East (D) Northeast
Sol. (D) The movements of the person are from A to F, as shown in fig. Clearly, the final position is F
which is to the Northeast of the starting point A.

Ex.8 The town of Paranda is located on Green lake. The town of Akram is West of Paranda. Tokhada
is East of Akram but West of Paranda. Kokran is East of Bopri but West of Tokhada and Akram. If
they are all in the same district, which town is the farthest West ?
(A) Paranda (B) Kokran (C) Akram (D) Bopri

   
Sol. (D) Bopri is the farthest West. Bopri Kokran Akram Tokhada Paranda

56
Ex.9 Sanjay went 70 metres in the East before turning to his right. he went 10 metres before turning to
his right again and went 10 metres from this point. From there he went 90 metres to the North.
How far was he form the starting point?
(A) 80 metres (B) 100 meters
(C) 140 metres (D) 260 metres
Sol. (B) The movement of Sanjay from A to E are as shown in Fig.
Now, AF = (AB - FB)
= (AB - DC) = (70 - 10) m = 60 m.
EF = (DE - DF) = (DE - BC)
= (90 - 10) m = 80m.

Required distance = AE = AF2 + EF2 = (60)2 + (80)2 = 100m

Ex.10 Starting from a point P, Sachin walked 20 metres towards South. He turned left and walked 30
metres. He then turned left and walked 20 metres. he again turned left and walked 40 metres and
reached a point Q. How far and in which direction is the point Q from the point P ?
(A) 20 metres West (B) 10 metres East (C) 10 metres West (D) 10 metres North

Sol. (C) The movements of sachin are as shown in figure


clearly, distance from starting point to final position

Ex.11 Shanshikant walks northwards. After a while, he turns to his right and cover a distance of 2 kms.
Then turns to his left and cover a distance of one kms, then he turns to his left again. In which
direction is he moving now ?
(A) North (B) South (C) East (D) West

Sol. (D) According to the given information :


Hence he is moving the West direction finally

57
PRACTICE EXERCISE

1. One evening before sunset two friends Amit and Sunit were talking to each other face to face. If
Sunil’s shadow was exactly to his left side, which direction was Amit facing ?
(A) North (B) South (C) West (D) Data inadequate

2. A postman was retuning to the post office which was in front of him to the North. When the post
office was 100 meters away from him, he turned to the left and moved 50 metres to deliver the
last letter at shantivilla. He then moved in the same direction for 40 metres, turned to his right and
moved 100 metres. How many metres was he away from the post office ?
(A) 0 (B) 90 (C) 150 (D) 100

3. Two buses start from the opposite points of a main road, 150 kms apart. The first bus runs for 25
kms and takes a turn right and runs for 15 kms. It then, turns left and runs for another 25 kms and
takes the direction back to reach the main road, In the meantime, due to a minor breakdown, the
other but has run only 35 kms along the main road What would be the distance between the two
buses at this point ?
(A) 75 kms (B) 870 kms (C) 65 kms (D) 85 kms

4. A man is facing west. He turns 450 in the clockwise direction and then another 1800 in the same
direction and then 2700 in the anticlockwise direction. Which direction is he facing now ?
(A) South (B) North-West (C) West (D) South-West

5. A started from a place. After walking for a kilometer, he turns to the left, then walking for a half
km. he again turns to left. Now, he is going Eastward direction. In which direction, did he originally
start ?
(A) West (B) East (C) South (D) North

6. Form point P, Akshay starts walking towards East. After walking 30 metres, he turns to his right
and walks 10 metres. He then turns to his right and walks for 30 metres. He again turns to his
right and walks 30 metres. He again turns to his right and walks 30 metres. How far is he from
Point P and in which direction ?
(A) Point P itself (B) 10 metres North (C) 20 metres West (D) 20 metres North

7. A walks 10 metres towards East and then 10 metres to his right. Then every time turning to his
left, he walks 5, 15 and 15 metres respectively. How far is he now from his starting point ?
(A) 5 metres (B) 10 metres (C) 15 metres (D) 20 metres

8. A and B start from a fixed point. A moves towards North and after walking 3 Kms turns to his right
and covers 4 Kms. B moves towards West and walks 5 Kms and then turns to his right and walks
3 Kms. Now how far are A and B from each other ?
(A) 1 Kms (B) 5 Kms (C) 8 Kms (D) 9 Kms

58
9. A person starts towards South direction. Which of the following order of directions will lead him to
East direction ?
(A) right, right, right (B) left, left, left (C) left, right, right (D) left, right, left

10. If I stand on my head with my face pointing Northwards, in what direction will my right-hand point
?
(A) East (B) West (C) North (D) South

11. The time on the watch is quarter to three. If the minute-hand points to North-East, in which
direction does the hour hand point ?
(A) South-West (B) South-East (C) North-West (D) North-East

12. A and B start walking from the same point. A foes North and covers 3 km; then turns right and
covers 4 km. B goes west and covers 5 km, then turns right and covers 3 kms. How far apart are
they from each other ?
(A) 10 km (B) 9 km (C) 8 km (D) 5 km

13. A and B start walking in opposite direction. A walked 5 km, B 6 km. Thereafter both turned to their
right and walked 2 km. They turned to right again and walked 3 km, again turned to right and
walked 2 km. How much distant apart are they from each other ?
(A) 2 km (B) 13 km (C) 3 km (D) 5 km

14. A watch reads 4 : 30. If the minute-hand points to East, in which directions does the hour-hand
point ?
(A) North-East (B) South-East (C) North-West (D) North

15. L is to South-West of K, M is to the East of L and South-East of K and N is to the North of M in


line with LK. In which direction of K is N located ?
(A) North (B) East (C) South-East (D) North-East

16. If South-East becomes North, North-East becomes West and so on, what will South become ?
(A) North-East (B) South - West (C) South (D) Northwest

17. I run along the sides of a square field ABCD where C is to the North-East of A and D is to the
South-East of B. Starting From A in anti-clockwise direction, in which direction shall I be running
after crossing C ?
(A) East (B) West (C) North (D) South

59
18. Shehnaz wants to go to the School. She starts from her home which is in North and comes to the
crossing. The road to her left ends in a park and straight ahead is the office complex. In which
direction is the School ?
(A) East (B) North (C) West (D) South

Directions : (19 to 23) Read the following statements and choose the correct alternative.
(i) A is a north of E and west of C.
(ii) B is north of A and west of P.
(iii) D is south and east of A.
(iv) E is north of F and east of D.
(v) F is north D and west of A.
(vi) C is south of F and west of D.

19. Which of the towns is furthest to the north west ?


(A) A (B) B (C) C (D) E

20. Which of the following must be both north and east of F ?


I. A II. C III. E
(A) II only (B) III only (C) I and II (D) I and III

21. Which of the following towns must be situated both south and west of at least one other town ?
(A) A and E (B) A and F (C) B and F (D) C, D and E

22. Which of the following statements, if true, would make the information in the numbered
statements more specific ?
(A) C is north of D (B) E is north of D (C) A is east of B (D) C is east of F

23. I run along the sides of square field ABCD where C is to the north east of A and D is to the south
east of B. Starting from A in anticlock wise direction, in which direction shall I be running after
crossing B ?
(A) North (B) South (C) East (D) West

Directions : (24 to 26) The following questions are based on the diagram given below showing
four persons stationed at the four corners of a square piece of plot as shown.

24. A starts crossing the field diagonally. After walking half the distance, he turns right, walks some
distance and turns left. Which direction is A facing now ?
(A) North-east (B) North-west (C) North (D) South-east

60
25. From the original position given in the above figure. A and B move one arm length clockwise and
then cross over to the corner diagonally opposite ; C and D move one arm length anti-clockwise
and cross over the corner diagonally opposite. The original configuration ADBC has now changed
to
(A) CBDA (B) BDAC (C) DACB (D) ACBD

26. From the original position, B and D move one and a half length of sides clockwise and
anticlockwise respectively. Which one of the following statements is true?
(A) B and D are both at the midpoint between A and C
(B) D is at the midpoint between A and C, and B at the corner originally occupies by C.
(C) B is at the midpoint between A and C, and D at the corner originally occupied by A.
(D) B and D are both at the midpoint between A and D.

ANSWERS

Que. 1 2 3 4 5 6 7 8 9 10 11 12 13
Ans. A B A D A D A D A B A B D
Que. 14 15 16 17 18 19 20 21 22 23 24 25 26
Ans. A D A B C B D B A B B A A

61
ANALOGY

Analogy means ‘Similarity’. A particular relationship is given and another similar relationship has to
be identified from the alternative provided.
KINDS OR RELATIONSHIPS :
Study & topic Relationship :
Some Examples :
1. Botany : Study of Plants 2. Zoology : Animals 3. Seismology : Earthquakes
4. Astrology : Future 5. Pathology : Diseases 6. Cardiology : Heart
7. Astronomy : Planets 8. Tectonics : Buildings 9. Taxidermy : Stuffing (Animals)
10. Penology : Punishment 11. Cytology : Cells 12. Geology : Earth
13. Archaeology : Artifacts 14. Ecology : Environment 15. Anthology : Collection of Poems
16. Trigonometry : Triangles 17. Menstruation : Area 18. Ornithology : Birds
19. Onomatology : Names 20. Ontology : Reality 21. Ethnology : Human Races
22. Herpetology : Amphibians (Anthropology : Man) 23. Paleography : Writings
24. Ichthyology : Fish (es) 25. Semantics : Language 26. Nephrology : Kidney
27. Concology : Shells 28. Hematology : Blood 29. Mycology : Fungi
30. Oology : Eggs 31. Virology : Viruses 32. Craniology : Skill
33. Entomology : Insects 33. Occultism Supernatural 37. Taxonomy : Classification
38. Orogaphy : Mountains 39. Selenography : Moon 40. Eccrinology : Secretions
41. Histology : Tissues 42. Nidology : Nests 43. Phycology : Algae
44. Bryology : Bryophyte

Worker & Tool Relationship :

Ex. Laborer : Spade


Spade is a tool used by a Laborers.
Some more examples -
1. Carpenter : Saw 2. Wood cutter : Axe 3. Blacksmith : Anvil
4. Soldier : Gun 5. Tailor - Needle 6. Chef : Knife
7. Framer : Plough 8. Author : Pen 9. Warrior : Sword
10. Sculptor : Chisel 11. Mason : Plumb line 12. Doctor : Stethoscope
13. Gardner : Harrow 14. Surgeon : Scalpel 15. Cobbler : Awl
16. Lumberjack : Axe 17. Painter : Brush 18. Violinist : Bow
19. Barber : Scissors 20. Butcher : Chopper
21. Astronomer : Telescope 22. Jockey : Tack

62
Tool & Action Relationship :
Ex. Needle : Saw
1. Knife : Cut 2. Gun : Short 3. Pen : Write
4. Microscope : Magnify 5. Spanner : Grip 6. Sword : Slaughter
7. Filter : Purity 8. Spade : Dig 9. Mattock : Dig
10. Steering : Drive 11. Spoon : Feed 12. Chisel : Carve
13. Axe : Grind 14. Shield : Guard 15. Loudspeaker : Amplify
16. Auger : Bore 17. Oar : Row 18. Shovel : Scoop

Worker and Working place :


Ex. Chef : Kitchen
Chef works in a kitchen
1. Farmer : Field 2. Warrior : Battle field 3. Engineer : Site
4. Sailor : Ship 5. Pilot : Cockpit 6. Beautician : Parlor
7. Actor : Stage 8. Mechanic : Garage 9. Lawyer : Court
10. Scientist : Laboratory 11. Waiter : Restaurant 12. Gambler : Casino
13. Servant : House 14. Worker : Factory 15. Umpire : Pitch
16. Teacher : School 17. Artist : Theatre 18. Doctor : Hospital
19. Clerk : Office 20. Driver : Cabin 21. Painter : Gallery
22. Grocer : Shop

Worker & Product :


Ex. Poet : Poem
Poet writes poem :
Ex. Chef : Food
Chef makes food
1. Farmer : Crop 2. Author : Book 3. Gold Smith : Ornaments
4. Cobbler : Shoes 5. Editor : Newspaper 6. Hunter : Prey
7. Carpenter : Meat 8. Butcher : Meat 9. Judge : Justice
10. Producer : Film 11. Architect : Design 12. Tailor : Clothes
13. Dramatist : Play 14. Choreographer : Ballet 15. Teacher : Educations
16. Mason : Wall

Product and Raw Material :


Ex. Cloth : Fibre (Cloth is made of Fibre)
1. Paper : Pulp 2. Book : Paper 3. Jewelry : Gold
4. Oil : Seed 5. Road : Asphalt 6. Sack : Jute
7. Metal : Ore 8. Fabric : Yarn 9. Pullover : Wool
10. Furniture : Wood 11. Butter : Milk 12. Wine : Grapes
13. Omelette : Egg 14. Rubber : Latex 15. Wall : Brick
16. Shoes : Leather 17. Prism : Glass 18. Linen : Flax
19. Jaggery : Sugarcane

63
Instrument & Measurement :
Ex. Ammeter : Current
1. Scale : Length Scale in an Instrument used to measure length.
2. Balance : Mass 3. Thermometer : Temperature 4. Odometer : Speed
5. Hygrometer : Humidity 6. Screw gauge : Thickness 7. Seismograph : Earthquake
8. Anemometer : Wind vane 9. Taseometer : Strains 10. Rainguage : Rain
11. Barometer : Pressure 12. Sphymonometer : Blood Pressure

Quantity & Unit :


Ex. Time : Seconds
Seconds in the unit of Time.
1. Force : Newton 2. Length : Meter 3. Energy : Joule
4. Work : Joule 5. Current : Ampere 6. Volume : Litre
7. Power : Watt 8. Potential : Volt 9. Mass : Kilogram
10. Pressure : Pascal 11. Area : Hectare 12. Temperature : Degrees
13. Resistance : Ohm 14. Angle : Radians 15. Magnetic field : Oersted
16. Conductivity : Mho 17. Luminosity : Candela

Animal & Young ones :


Ex. Dog : Puppy (Puppy is the young one of Dog)
1. Lion : Cub 2. Man : Child 3. Hen : Chicken
4. Sheep : Lamp 5. Cow : Calf 6. Cat : Kitten
7. Duck : Duckling 8. Horse : Pony/Calf 9. Insect : Larva
10. Stallion : Colt 11. Butterfly : Caterpillar 12. Frog : Tadpole

Male & Female :


Ex. Tiger : Tigress
Tigress is Female tiger
1. Son : Daughter 2. Gentleman : Lady 3. Nephew : Niece
4. Drone : Bee 5. Dog : Bitch 6. Stage : Doe
7. Sorcerer : Sorceress 8. Horse : More 9. Lion : Lioness

Word & Synonym :


Ex. Vacant : Empty (Empty means almost the same as Vacant)
1. Substitute : Replace 2. Blend : Mix 3. House : Home
4. Solicit : Request 5. Flaw : Defect 6. Fierce : Violent
7. Dearth : Scarcity 8. Ban : Prohibition 9. Mend : Repair
10. Assign : Allot 11. Abduct : Kidnap 12. Sedate : Calm
13. Alight : Descend 14. Pressure : Assume 15. Presage : Predict
16. Fallacy : illusion 17. Brim : Edge 18. Dissipate : Squander
19. Haughty : Proud 20. Dissipate : Squander
Word & Antonym :

64
Ex. Good : Bad
1. Cruel : Kind 2. Best : Worst 3. Sink : Float
4. Strong : Weak 5. Initial : Final 6. Start : End
7. Ignore : Notice 8. Advance : Retreat 9. Create : Destroy
10. Gentle : Harsh 11. Gradual : Abrupt (Sudden) 12. Condense : Expand
13. Deep : Shallow 14. Affirm : Deny 15. Kindle : Extinguish
16. Mourn : Rejoice 17. Cordial : Hostile 18. Kindle : Extinguish
19. Chaos : Peace 20. Fresh : Stale 21. Lend : Borrow

Words & Intensity :


Ex. Quarrel : War
1. Anger : Rage 2. Kindle : Burn 3. Error : Blunder
4. Wish : Desire 5. Sink : Drown 6. Famous : Renowned
7. Unhappy : Sad 8. Crime : Sin 9. Refuse : Deny
10. Moist : Drench

SIMPLE ANALOGY :
Directions : (1 to 3) In the following questions, choose the words that show the same relationship
as given in the each questions.
Ex.1 Flower is to a Bouquet as Minister is to a.
(A) Voter (B) Cabinet (C) Constituency (D) Department
Sol. (B) Second word Bouquet is group of first word ‘Flower’. In the same manner Cabinet is a group
of Ministers.

Ex.2 Hour is related to Second in the same way as Tertiary is related to


(A) Ordinary (B) Secondary (C) Primary (D) Intermediary
Sol. (C) Second is the third positions after Hour in time measurement. Likewise Tertiary is the third
position after Primary in he other of ranking.

Ex.3 Sports is related to Logo in the same way as Nation is related to


(A) Emblem (B) Animal (C) Ruler (D) Anthem
Sol. (A) The symbol Logo is related to Sports. Likewise Emblem is related to Nation.

65
LETTER ANALOGY :
In letter analogy questions, the question pair and answer pair consists of letter. You have to
examine the question pair and find the relationship between them and choose the answer pair
that contains the same analogy or relationship as in the question pair.

Directions : (4 to 13) In each of the following questions, there are two terms to the left of the sign
:: which are related in some way. Obtain the same relationship between the term to the
right of the sign :: from one of the flour alternatives given under it.

Ex.4 ef : jk :: no : ?
(A) dc (B) gi (C) ml (D) tu
Sol. (D) In the questions pair ‘ef : jk’. The letters of the first term ‘ef’ are in natural alphabetic
sequence. So is the second term ’jk’. The letter ‘no’ are also in the natural alphabetic sequence.

Ex.5 FG : LM :: ? : ?
(A) NO : TO (B) HI : RS (C) CH : KL (D) DE : BA
Sol. (A) Examine the questions pair ‘FG : LM’. The relationship is that the lettes are in alphabetic order
and five letters are skipped between terms.

Ex.6 LXNU : NYPV : QTBR : ?


(A) RUSD (B) SDSU (C) SUDS (D) RSUD
Sol. (C) Second term is obtained from the first by moving its first and third letters two steps forward
while the second and fourth letters one step forward.

Ex.7 MANTEL : NAMLET :: VANITY : ?


(A) NAVYIT (B) NAVYTI (C) NAVIYI (D) AVNTIY
Sol. (B) Group of three letters is reversed.

Ex.8 TUEDAY : UUFSCAX :: SQUAREE : ?


(A) TQUASED (B) TQVASED (C) TQVAQED (D) TQVARED
Sol. (C) Sequence is + 1, 0, +1, 0, -1, 0, -1

Ex.9 AEZ : EIY :: IOX : ?


(A) UYZ (B) AEZ (C) EIX (D) OUW
Sol. (D) Each term has two vowels in the beginning, and the first letter from backward sequence.
Hence AE (vowels) Z, El (vowels) Y etc.

Ex.10 ECF : FDG :: IEH : ?


(A) OFJ (B) OFI (C) GHI (D) LMN
Sol. (B) Each item stars with a vowel which maintains the sequence of AEIOU as is seen from other
items. After a vowel, 2 letters follow, of which 2 intervening consecutive letters are skipped. i.e.
C(DE) F, D(F) G, E(FG) H and F (GH) I.

66
Ex.11 CG : EI :: FJ : ?
(A) JK (B) IJ (C) LM (D) GK
Sol. (D) Letter groups consist of 2 letters in alphabetic order skipping 3 letters immediately following :

Ex.12 DFHJ : LNPR :: ? : BDFH


(A) VXZT (B) UVXZ (C) TXVZ (D) TVXZ
Sol. (D) All the letters of the second term are moved eight steps backward to obtain the first term.

Ex.13 DULC : EVMD :: ? : GXOF


(A) FWNE (B) HNWE (C) HWNE (D) FUEN
Sol. (A) First term is obtained from the second by moving all its letters one step backward.

WORD ANALOGY :

Ex.14 India Gate : Delhi : ________ : _________


(A) Chicago : USA (B) Albany : New York
(C) Agra : Taj Mahal (D) Chandigarh : Rock Garden
Sol. (B) India Gate is in Delhi, Albany is in New York.

Ex.15 PUNJAB : AMRITSAR :: ______ : ______


(A) Golden Temple : Amritser (B) Moscow : Russia
(C) India : Asia (D) Agra : Taj Mahal
Sol. (D) Amritser is in Pujab, Taj Mahal is in Agra

Direction : (16) In each of the following questions, two words indicated by I and Ii have been left
out. The correct word to come in place of I is given as one of the four alternatives against I
and the correct word to come in place of Ii is given as one of the four alternatives against
II. Read with the correct words, there is some relationship between the two words to the
left of the sign (: :) and the same relationship obtains between the two words to the right of
the sign (: :) The correct combination is given as one of the four alternatives (a), (b), (c) and
(d). Find the correct combination in each case.

Ex.16 I : Melt : : Bright : II


I. (a) Liquid (b) Ice (c) Heat (d) Freeze
II. (P) Dull (Q) Dazzle (R) Light (S) Colour
(A) aS (B) bR (C) cQ (D) dP
Sol. (d) The words in each pair are antonyms of each other.

Directions : (17 to 18) In each of the following questions, a group of three interrelated words is
given. Choose a word from the given alternates, that belongs to the same group.

Ex.17 Marble : Slate : Gneiss


(A) Quartzite (B) Limestone (C) Coal (D) Sandstone
Sol. (A) All are metamorphic rocks.

67
Ex.18 Pituitary : Thyroid : pancreas
(A) Adrenal (B) Heart (C) Liver (D) Kidney
Sol. (A) All are endocrine glands.

Directions : (19 to 20) Three words in bold letters are given in each question, which have
something in common among themselves. Out of the four given alternatives, choose the
most appropriate de-scription about these three words.

Ex.19 Analects : Zend Avesta : Torah


(A) These are places of worship (B) These are three sects of Muslims
(C) These are names of religions (D) These are names of religious books.
Ans. (D)

Ex.20 Hiss : Hoot : Trumpet


(A) They are sounds made by certain creatures
(B) They are joyous cries of children
(C) They are sound made by war-instruments.
(D) The terms are used in connection with under-world activities.
Ans. (A)

NUMBER ANALOGY :

Directions : (21 to 23) In each of the following questions, there is a certain relation between two
given number on one side of :: and one number is given on another side of :: while another
number is to be found from the given alternatives, having the same relation with this
number as the numbers of the given pair bear. Choose the best alternative.

Ex.21 7584 : 4251 :: 4673 : ?


(A) 1367 (B) 1340 (C) 1531 (D) None of these

Ex.22 225 : 257 :: 289 : ?


(A) 301 (B) 316 (C) 320 (D) 325

Ex.23 5 : 18
(A) 30 : 96 (B) 21 : 66 (C) 19 : 61 (D) 11 : 35
Sol. (B) The relationship is x : (3x + 3)

68
Directions : (24 to 26) In each of the following questions, choose one number which is similar to
the numbers in the given set.

Ex.24 Given set : 192, 282, 372


(A) 453 (B) 461 (C) 236 (D) 425
Sol. (A) In all the numbers, the sum of digits is 12 and the largest digit lies in the middle.

Ex.25 Given set : (8, 15, 24)


(A) (6, 13, 21) (B) (10, 17, 28) (C) (11, 18, 27) (D) (13, 20, 32)
Sol. (C) In each set, 2nd number = 1st number + 7 ; 3rd number = 2nd number + 9.

Ex.26 Given set : (8, 3, 2)


(A) (10, 6, 5) (B) (63, 8, 3) (C) (95, 24, 5) (D) (168, 15, 4)

Sol. (B) each set, 1st number = (2nd number)2 - 1 ; 2nd number = (3rd number)2 - 1.

PRACTICE EXERCISE

Directions : (1 to 4) in the Following questions, choose the words that show the same relationship
as given in the each questions.

1. Bank is related to Money in the same way as Transport is related


(A) Goods (B) Road (C) Terrace (D) Floor

2. What is related to Taka in the same way as Lira is related to Italy ?


(A) Pakistan (B) Jordan (C) Mexico (D) Bangladesh

3. Needle is related to Clock as Wheel is related to _____


(A) Drive (B) Vehicle (C) Circular (D) Move

4. Boat is related to Oar in the same way as Bicycle is related to


(A) Road (B) Wheel (C) Seat (D) Paddle

69
Directions : (5 to 11) In each of the following questions, there are two terms to the left of the sign
:: which are related in some way. Obtain the same relationship between the term to the
right of the sign :: from one of the four alternatives given under it.

5. ? : CEIG :: LNRP : OKUM


(A) FELD (B) ZHFJ (C) FHFJ (D) ABLD

6. KLQM : CFMK :: NRPT : ?


(A) FLLR (B) HIJH (C) FLTM (D) RLTM

7. LJPN : KMOQ :: ? : XVTZ


(A) YSUV (B) SUWY (C) VTWY (D) YSUW

8. APOC : ? :: ITSK : MVUN


(A) DRQH (B) ERQF (C) EQRG (D) DQRH

9. AZB : BYC :: CXD : ?


(A) DWE (B) DEF (C) DFG (D) DMN

10. ABCD : WXYZ :: EFGH : ?


(A) STUV (B) TSUV (C) STUE (D) STVU

11. ACEG : ? :: BDFH : KMOQ


(A) LMNO (B) JLNP (C) JNLO (D) JLON

Directions : (12 to 13) In each of the following questions, two word indicated by I and Ii have been
left out. The correct word to come in place of I is given as one of the four alternatives
against I and the correct word to come in place of Ii is given as one of the four alternatives
against II. Read with the correct words, where is some relationship between the two words
to the left of the sign (::) and the same relationship obtains between the two words to the
right of the sign (::). The correct combination is given as one of the four alternatives (a),
(b), (c) and (d). Find the correct combination in each case.

12. I : Water :: Thermometer : II


I. (a) Humidity (b) Rain (c) Pitcher (d) Evaporation
II. (P) Temperature (Q) Mercury (R) Doctor (S) Fever

(A) aS (B) cQ (C) dP (D) bR

70
13. I : Flower :: Milky way : II
I. (a) Garden (b) Plant (c) Fruit (d) petals
II. (P) Galaxy (Q) Star (R) Sky (S) Planet
(A) bP (B) dR (C) aQ (D) cS

Directions : (14) Question consists of a pair of numbers that have a certain relationship to each
other, followed by four other pairs of numbers given as alternatives. Select the pair in
which the numbers are similarly related as in the given pair.

14. 11 : 1210
(A) 6 : 216 (B) 7 : 1029 (C) 8 : 448 (D) 9 : 729

Direction : (15) Questions consists of a particular pattern. Find the pattern and answer the
questions.

15. Given set : 992, 733, 845, 632


(A) 114 (B) 326 (C) 425 (D) 236

Direction : (16) In each of the following questions, choose that set of numbers form the alternative
sets. That is similar to the given set ?

16. Given : (246, 257, 358)


(A) (144, 235, 325) (B) (143, 253, 246) (C) (273, 365, 367) (D) (233, 343, 345)

Directions : (17 to 18) In each of the following questions, a group of three interrelated words in
given. Choose a word from the given alternatives that belongs to the same group.

17. Potato : Carrot : Reddish


(A) Tomato (B) Spinach (C) Sesame (D) Groundnut

18. Basket : Pail : Pan


(A) Spoon (B) Bowl (C) Fork (D) Knife

71
Directions: (19 to 20) three words in bold letters are given in each question, which have
something in common among themselves. Out of the four given alternatives, choose the
most appropriate description about these three words.

19. Spinach : Fenugreek : Celery


(A) These are cactus plant (B) These are wild flowers
(C) These are wild plants (D) These are leafy vegetables
20. Petrol : Phosphorus : Cooking gas
(A) They are fuels (B) They are highly inflammable
(C) They can’t be sold without permit (D) India has to import them

Directions : (21 to 27) In the following question, choose the pair/group of words that show the
same relationship as given at the top of every pair/group.

21. Manager : Cabin


(B) Driver : Train (B) Captain : Desk (C) Pilot : Cockpit (D) Servant : Hospital

22. Aeroplane : Hanger


(A) Train : yard (B) Train : Plant form (C) Train : Rail (D) Train : Railway station

23. Engineer : Machine


(A) Doctor : Disease (B) Doctor : Medicine (C) Doctor : Hospital (D) Doctor : Body

24. Mosquito : Malaria : :


(A) Tobacco : Cancer (B) Road : Accident (C) Housefly : Food (D) Soil : Erosion

25. Light : Ray :: Sound : ?


(A) Hear (B) Wave (C) Audio (D) Pitch

26. Paisa - Rupee, centimeter - Metre, Kilogram - ?


(A) Metric tonne (B) Hectogram (C) Quintal (D) Gram

27. Water : Oxygen


(A) Helium : Nitrogen (B) Salt : Sodium (C) Tree : Plant (D) Food : Hunger

72
Direction : (28 to 29) Find out the correct words from the options to fill in the blanks. The world
which is in some way related to the word on the right as well as to the word on its left is
the correct answer.

28. Medicine __spacecraft


(A) Effective (B) Advanced (C) Capsule (D) Homeopathy

29. Money __ River


(A) Flow (B) Liquid (D) Dam (D) Bank

ANSWERS

Que. 1 2 3 4 5 6 7 8 9 10 11 12 13 14 15
Ans. A D B D B A D B A A B B C C C
Que. 16 17 18 19 20 21 22 23 24 25 26 27 28 29
Ans. C D B D B C A D A B C B B D

73
CLASSIFICATION

CLASSIFICATION :

‘Classification’ means ‘to assort the items of a given group on the basis of certain common quality
they posses and then spot the stranger out’/

Directions (1 to 5) In each of the following questions, five words are given, out of which four are
same in one way and the fifth one is different from others. Select the odd one.

Ex.1 (A) Sun (B) Moon (C) Venus (D) Mars


(E) Earth
Sol. (B) All the terms except Moon are related to the Solar system.

Ex.2 (A) Green (B) Violet (C) Brown (D) Yellow


(E) Orange
Sol. (C) Except Brown all the colours are present in rainbow.

Ex.3 (A) Silk (B) Fur (C) Milk (D) leather


(E) Rubber
Sol. (E) Only Rubber is the three product.

Ex.4 (A) Milk (B) Syrup (C) Squash (D) Tea


(E) Cake
Sol (E) All other are the drinks.

Ex.5 (A Conscience (B) Morality (C) Conduct (D) Will-power


(E) Weight
Sol. (E) All other terms are used to represent human behavioral personality factors.

Directions : (6 to 9) In each of the following questions four out of five alternatives contain
alphabet placed in a particular from. Find the one that does not belong to the group.

Ex.6 (A) NKMJ (B) FCEB (C) URTQ (D) KHJG


(E) TQRP
Sol. (E) In all other groups there is a gap of one letters in the alphabet between second and third letter.

74
Ex.7 (A) DW (B) GT (C) KP (D) FR
(E) HS
Sol. (D) In all other pairs of words first and second letters are equidistant from the beginning and end
respectively in the alphabetical series.

Ex.8 (A) A8C (B) D22G (C) H42M (D) B36P


(E) F34J
Sol. (E) In all other groups number between first and second letter is twice the sum of positions of first
and last letters in the alphabet.

Ex.9 (A) KQ14 (B) AY13 (C) MT11 (D) GW15


(E) LZ19
Sol. (C) In all other groups number at the end is half of the positions of sum of first and second letters
in the alphabet.

Directions : (10 to 14) In the following question, numbers given in four out of the five alternatives
have some relationship. You have to choose the one which does not belong to the group.

Ex.10 (A) 3 : 8 (B) 6 : 35 (C) 7 : 50 (D) 1 : 0


(E) 9 : 80
Sol. (C) In other numbers second number is one less than the square of first number.

Ex.11 (A) 21 : 24 (B) 28 : 32 (C) 14 : 16 (D) 70 : 80


(E) 54 : 62
Sol. (E) The ratio among the number is 7 : 8

Ex.12 (A) 4 (B) 8 (C) 16 (D) 9


(E) 25
Sol. (B) All other numbers are square of natural numbers.

Ex.13 (A) 22 : 0 (B) 24 : 12 (C) 23 : 5 (D) 18 : 63


(E) 24 : 18
Sol. (E) Second number is the difference of the square of digits of first number.

Ex.14 (A) 43 (B) 53 (C) 63 (D) 73


(E) 83
Sol. (C) All other numbers are prime numbers.

75
PRACTICE EXERCISE

Directions : (1 to 23) In the following questions, three out of the four alternatives are same in a
certain way and so form a group. Find the odd one that does not belong to the group.

1. (A) Gold (B) Silver (C) Bronze (D) Iron

2. (A) Yen (B) Lira (C) Dollar (D) Ounce

3. (A) Huge (B) Tiny (C) Heavy (D) Small

4. (A) Teeth (B) Tongue (C) Palate (D) Chin

5. (A) Silk (B) Cotton (C) Nylon (D) Wool

6. (A) Triangle (B) Tangent (C) Square (D) Rhombus

7. (A) Lion-Deer (B) Cat-Mouse (C) Hawk-Pigeon (D) Pig-Piglet

8. (A) Work-Leisure (B) Day-Night


(C) Expedite-procrastinate (D) Frequently-Always

9. (A) April (B) May (C) July (D) September

10. (A) DBF (B) HFK (C) NLP (D) XVZ

11. (A) MrW (B) ChN (C) KpU (D) BgL

12. (A) DFHB (B) KMOJ (C) PRTN (D) XZBV

13. (A) ROQP (B) KHJI (C) VSUT (D) JHIG

14. (A) ACE (B) MOQ (C) RTV (D) UWY

15. (A) BDI (B) KMR (C) PRW (D) FHN

76
16. (A) 232 (B) 362 (C) 661 (D) 284

17. (A) 488 (B) 929 (C) 776 (D) 667

18. (A) 9 : 80 (B) 1 : 0 (C) 12 : 13 (D) 10 : 91

19. (A) 4, 6, 10, 7 (B) 4, 12, 20, 28 (C) 1, 3, 5, 7 (D) 2, 6, 10, 14

20. (A) 22 : 44 (B) 39 : 981 (C) 45 : 1625 (D) 24 : 464

21. (A) 385 (B) 572 (C) 671 (D) 427

22. (A) 27 (B) 125 (C) 1321 (D) 729

23. (A) 9 - 27 (B) 15 - 45 (C) 10 - 30 (D) 20 - 60

ANSWEERS

Que. 1 2 3 4 5 6 7 8 9 10 11 12
Ans. C D C D A B D D A B B B
Que. 13 14 15 16 17 18 19 20 21 22 23
Ans. D C D A D D A D D C A

77
LOGICAL VENN DIAGRAM

1. An object is called a subset of another object, if former is a part of latter and such relation is shown
by two concentric circle.
(i) Pencil, Stationary (ii) Brinjal, Vegetable (ii) Chair, Furniture
It is very clear from the above relationship that one object is pat of other, and hence all such
relationships can be represented by figure below -

2. An object is said to have an intersection with another object, when two objects share some thing
in common.
(i) Surgeon, Males
(ii) Politician, Indian
(iii) Educated, Unemployed

All the three relationship given above have something in common as some surgeons can be male
and some female, some politician may be Indian and some may belong to other countries,
educated may be employed and unemployed as well. And all the three relationships can be
represented by figure above.

3. Two objects are said to be disjoint when neither one is subset of another nor they share anything
in common. In other words, totally unrelated object fall under this type of relationship
(i) Furniture, Car
(ii) Copy, Cloth
(iii) Tool, Shirt

78
It is clear from the above relationship hat both the objects are unrelated to each other, and hence
can be represented diagrammatically as shown in figure above.
From the above discussion we observe that representation of relationship of two objects is not
typical if students follows the above points. But representation of three objects diagrammatically
pose slight problem before the students. A variety of such relationship is being discussed in the
following examples.

Direction : (1 to 4) Each of these questions given blow contains three group of things. you are to
choose from the following five numbered diagrams, a diagram that depicts the correct
relationship among the three groups of thing in each question.

(A) (B) (C) (D)

Ex.1 Moon, Earth, Universe


Ex.2 India, Pakistan, Asia
Ex.3 Batsman, Cricket, Stick
Ex.4 Book, Pen, Pencil

Sol (1 to 4) :
1. Moon and Earth, are the parts of universe and therefore are subsets of universe and hence this
relationship is represented by diagram A).
2. India and Pakistan are the subsets of Asia. Hence, option (A) represents this relationship.
3. Batsman is a subset of Cricket and, Stick is something unrelated to Cricket, therefore, our answer
is (D).
4. Book, Pen, Pencil are neither subset of one another nor have anything in common. Therefore, our
answer is (C).
Ex.5 Which of the following diagram correctly represents the relationship among Tennis Fans, Cricket
players and Students.

(A) (B) (C) (D)

Sol. (A) From the relationship given in the question, we observe that each of the objects carries
something in commo0n to one another. A Tennis fan can be a cricket player as well as student.
Hence Diagram (A) represents this relationship.

79
Ex.6 Which of the following diagrams correctly represents the relationship among smokers, bidi
smokers, cancer patients.

(A) (B) (C) (D)

Sol. (B) Bidi smokers is a subset of smokers and cancer patient may be a smoker, bidi smoker and
non-smoker. Hence third object shares a common relationship with first and second object as
well.

Directions : (7 to 12) In the following diagram, three classes of populations are represented by
three figures. The triangle represents the school teachers, the square represents the
married persons and the circle represents the persons living in joint families.

Ex.7 Married persons living in joint families but not working as school teachers are represented by
(A) C (B) F (C) D (D) A

Ex.8 Persons who live in joint families, are unmarried and who do into work school teachers are
represented by
(A) C (B) B (C) E (D) D

Ex.9 Married teachers living in joint families are represented by


(A) C (B) B (C) D (D) A

Ex.10 School teachers who are married but do not live in joint families are represented by
(A) C (B) F (C) A (D) D

Ex.11 School teachers who are neither married nor do live in joint families are represented by
(A) F (B) C (C) B (D) A

Sol. (7 to 11)

7. (C) Married persons living in joint families are presented by the region common to the square and
the circle i.e., D and B. But, according to the given conditions, the persons should not be school
teachers. So, B is to be excluded. Hence, the required condition is denoted by region D.

80
8. (C) Persons living in joint families are represented by the circle. According to the given conditions,
the persons should be unmarried and not working as school teachers. So, the region should not
be a part of either the square of the triangle. Thus, the given conditions are satisfied by the
regions E.

9. (B) Married teachers are represented by the region common to the square and the triangle i.e., B
and C. But, according to the given conditions, the persons should be living in joint families. So, the
required regions should be a part of the circle. Since B lies inside the circle, so the given
conditions are satisfied by the persons denoted by the regions B.

10. (A) As in the above questions, married teachers are represented by B and C. Bit. here, the given
conditions lay down that the persons should not be living in joint families. So, the required regions
should lie outside the circle. Since C lies outside the circle, so the given conditions are satisfied by
the person denoted by the regions C.

11. (A) School teacher are represented by the triangle. But according to the given conditions, persons
are neither married nor do they live in joint families. So, the regions should to be a part of either
the square or the circle. Such a region is F.

PRACTICE EXERCISE

Directions : (1 to 2) Each questions below has three items having certain relationship among
them. The same relationship is expressed by sets of circles, each circle representing one
item irrespective of its size. Match the item with right set of circles.

(A) (B) (C) (D)

1. Women, Married persons, Wives who work.


(A) A (B) C (C) D (D) B

2. Computer skilled, Graduates, Employed.


(A) C (B) D (C) B (D) A

81
Directions : (3 to 4) Out of the four alternatives in each of the following questions, three
alternatives are such that the three words in each are related among themselves in one of
the five ways represented by (A), (B), (C), (D) and (E) below, And one of the alternative
represented a relationship which is not represented by any of the figures given below. The
relationship that complies this condition is your answer.

(A) (B) (C) (D) (E)

3. (A) Animal, Mammal, Cow (B) Colour, Cloth, Merchant


(C) Colour, Red, Blue (D) Male, Horse, Mare

4. (A) Periodicals, Weekly, Book (D) Mineral, Copper, Wood


(C) Doctors, Human beings, Married people (D) Army, Doctors, Engineers

Directions : (5 to 9) Study the figure below and answer the following questions.

5. Find out the number of families which have all the four things mentioned in the diagram.
(A) 40 (B) 30 (C) 35 (D) 20

6. Find out the number of families which have scoters


(A) 145 (B) 100 (C) 188 (D) 240

7. Find out the number of families which have V.C.R. and T.V. both
(A) 84 (B) 24 (C) 104 (D) 100

8. Find out the number of families which have only one thing, that is, either V.C.R. or T.C. or Scooter
or Maruti.
(A) 160 (B) 184 (C) 225 (D) 254

9. Find out the number of families which have T.V. and scooter both but have neither V.C.R. nor
Maruti.
(A) 15 (B) 30 (C) 4 (D) 50

82
Directions : (10 to 12) Each questions below contains three groups of things. You are to choose
from the following five numbered diagrams, the diagram that depicts the correct
relationship among the three groups of thins in each question.

(A) (B) (C) (D)

10. Vegetable, Apple, Spinach


11. Clever, Punctual, Poor
12. Copper, Cobalt, Silver

Directions : (13 to 15) in each of the following questions, select the diagram out of the five that
best represents the relationship among the items given in the questions.

(A) (B) (C) (D)

13. Doctor, Lawyer, Male


14. Man, Husband, Son
15. Female, Medicine, Physician

Directions :L (16 to 20) Out of the four alternatives in each of the following questions, three
alternatives are such that the three words in each are related among themselves in one of
the five ways represented by (A), (B), (C), (D) and (E) below, And one of the alternatives
represents a relationship which is not represented by any of the figures given below. The
relationship that complies this conditions is your answer.

(A) (B) (C) (D) (E)

83
16. (A) Army, General, Colonel (B) Boy, Student, Player
(C) Painter, Scholar, Table (D) Man, Typist, Peon
17. (A) Hen, Dog, Cat (B) Body, Ear, Mouth
(C) Bed, Ward, Nurse (D) Tiger, Animal, Carnivorous
18. (A) Mineral, Iron, Copper (B) Dean, Painter, Singer
(C) Seed, Leaf, Root (D) Piston, Engine, Wheel
19. (A) Direction, Engineer, Musician (B) Apple, Orange, Mango
(C) Fruit, Mango, Grass (D) Oxygen, Air, Water
20. (A) Atmosphere, Air, Oxygen (B) Boy, Girl, Student
(C) Man, Worker, Garden (D) Animal, Dog, Cat

Directions : (21 to 24) Read the following information carefully and answer the questions based
on them : The circle represents poor boys, the square educated boys, the triangle
represents the boys who are employed somewhere and the rectangle represents those
who help in the family business. Each section of the diagram is numbered.

21. Which number represents those poor boys who help in family business but are not educated or
employed elsewhere ?
(A) 2 (B) 3 (C) 4 (D) 5

22. Which number represents the group of educated poor boys who are employed somewhere but do
not help in family business ?
(A) 3 (B) 11 (C) 2 (D) None of these

23. Which section does number 12 represent ?


(A) Uneducated poor boys who do not help in family business
(B) Educated poor boys employed in service
(C) Uneducated boys who help in family business
(D) Educated poor boys who help in family business.

84
24. Which number represents that section of poor boys who are neither educated nor are in any
employment or have any family business ?
(A) 5 (B) 1 (C) 11 (D) 12

25. Which numbered space in the figure, represents doctors who are players as well as artists ?

(A) 2 (B) 3 (C) 6 (D) 7

Directions : (26 to 26) Study the following figure carefully and answer the questions :
The triangle represented doctors. The circle represents players and the rectangle
represents artists.

26. How many doctors are both players and artists ?


(A) 6 (B) 8 (C) 4 (D) 3

27. How many artists are players ?


(A) 30 (B) 29 (C) 25 (D) 17

28. How many artists are neither players nor doctors ?


(A) 29 (B) 30 (C) 22 (D) 8

29. How many doctors are neither players nor artists ?


(A) 17 (B) 30 (C) 8 (D) 19

ANSWERS

Que. 1 2 3 4 5 6 7 8 9 10 11 12 13 14 15
Ans. D B B C D C D C B C A B D C B
Que. 16 17 18 19 20 21 22 23 24 25 26 27 28 29
Ans. C A C B C D D A D D D C B A

85
THE CALENDAR

CONCEPT :
We are to find the day of the week on a mentioned date. Certain concept are defined as under.

ODD DAYS :
The no. of days exceeding the complete no. of weeks in a duration is the no. of odd days during
that duration.
ORDINARY YEAR :
An ordinary year has 365 days.
LEAP YEAR :
A leap year has 366 days. Every year which is divisible by 4 is called as leap year. For example
1200, 1600, 1992, 2004, etc. are all leap years.

IMPORTANT REMARK :
The First year of every century year ending in 00’s but not a multiple of 400 is not considered a
leap year. For example 900, 1000, 1100, 1300, 1400, 1500, 1700, 1800, 1900, 2100, are not leap
years.
COUNTING OF ODD DAYS :
(i) Every Ordinary years has 365 days = 52 weeks + 1 day ∴ (ordinary year has 1 odd day).
(ii) Every leap year 366 days = 52 weeks + 2 days ∴ (leap year has 2 odd days).
(iii) 100 years = 76 ordinary years + 24 leap years (The year 100 is not a leap year)
= 76 odd days + 2 × 24 odd days ⇒ 124 odd days.

124
= 5 (Remainder) = 05 odd days
7
Similarly, 200 years = 10 odd days = 03 odd days
15
300 years = = 01 odd day
7
20 + (1)
400 years = = 0 odd days {1 is added as 400 is a leap year}
7
Similarly, 800, 1200, 1600, 2000, 2400 years contain 0 odd days

86
COUNTING OF DAYS :
After counting the odd days, we find the day according to the no. of days → Sunday for 0 odd
day, Monday for 01 odd day and so on.

IMPORTANT NOTES :
(i) In an Ord. Year, First & last day of the year are the same.
Ex. If 1 Jan is Friday than 31 Dec. will also have Friday.
(ii) For a leap year, if first day is Monday than last day will be Tuesday for the same year.
(iii) Calendar year 1 Jan to 31 Dec. Financial year 1 April to 31 March.
(iv) The day on which calendar Started i.e., 1 Jan. 001 was Monday
(v) In a Leap year, February is of 29 days. In an ordinary year, February has 28 days.

Ordinary Year – 365 days Leap year – 366 days

January 31 January 31
February 28 February 29
March 31 March 31
April 30 April 30
May 31 May 31
June 30 June 30
Total 181 days Total 182 days
July 31 July 31
August 31 August 31
September 30 September 30
October 31 October 31
November 30 November 30
December 31 December 31
Total 184 days Total 184 days

ILLUSTRATIONS :
Ex.1 Find the day of the week on 16 January, 1969.
Sol. 1600 years have ‘0’ odd day ...........(A)
300 years have ‘1’ odd day ......... (B)
68 years have 17 leap years and 51 ordinary years.
Thus = (17 × 2 + 51 × 1) = 85 odd days
≅ ‘01’ odd day .....(C)
16 January has = ‘02’ odd days .....(D)
Adding (A) + (B) + (C) + (D),
We get, 0 + 01 + 01 + 02 = 04 odd days
Ans. Thursday

87
Ex.2 Find the day of the week on 18 July, 1776 (leap year)
Sol. Here 1600 years have ‘0’ odd day .......(A).
100 years have ‘5’ odd days ..........(B)
75 years = (18 leap years + 57 ordinary years)
= (18 × 2 + 57 × 1)
= 93 odd days
= (7 × 13 + 2) = ‘2’ odd days ........(C)
Now, the no. of days from 1st January to 18 July, 1776
= 182 + 18 = 200 days
= (28 × 7 + 4) days = ‘4’ odd days ...... (D)
Adding (A) + (B) + (C) + (D),
We get, 0 + 5 + 2 + 4 = 04 odd days
Ans. Thursday

Ex.3 On what dates of October, 1975 did Tuesday fall ?


Sol. For determining the dates, we find the days on 1st Oct, 1975.
1600 years have ‘0’ odd days ............(A).
300 years have ‘01’ odd days ..........(B)
74 years have (18 leap years + 56 ordinary years)
2 × 18 + 1 × 56 = 92 odd days
= ‘01’ odd days ......(C)
Days from 1 January to 1st Oct., 1975
st

1st Jan - 30 June + 1st July to 1st Oct.


181 + 31 + 31 + 30 + 1 = 274 days
= ‘01 odd days ....(D) (274/7 = 01 days)
Adding (A) + (B) + (C) + (D) = 0 + 01 + 01 + 01 = ‘03’ odd days
st
Ans. Wednesday (1 Oct), hence, 7, 14, 21, 28, Oct. will Tuesday fall

Ex04 Calendar for 1995 will serve for 2006, prove ?


Sol. The Calendar for 1995 and 2006 will be the same, if day on 1st January of both the years is the
same. This is possible only if the total odd days between 31st Dec. 1994 and 31st Des. 2005 is 0.
[one day before both the years as we want to know the day on 1st January of both the years i.e.
same]
During this period, we have
3 leap years and 08 ordinary years
(1996, 2000, 2004) (1995, 1997, 1998, 1999, 2001, 2002, 2003, 2005)
Total odd days = (2 × 3 + 1 × 8 ) = 14 = odd days (Thus Proved)

88
Ex.5 The year next of 1996 having the same Calendar will be -
Sol. 1996 1997 1998 1999 2000 2001 2002 2003
2 1 1 1 2
Total = 2 + 1 + 1 + 1 + 2 = 7 = 0 odd days
Hence, year 2001 will have the same calendar as year 1996.

Ex.6 Prove that last day of a century cannot be Tuesday, Thursday or Saturday.
Sol. 100 years have = 5 odd days ∴ Last days of Ist century is Friday
200 years have = 10 odd days ∴ Last day of IIst century is Wednesday
= 3 odd days
300 years have = 15 odd days ∴ Last day of IIIrd century is Monday
= 01 odd day
400 years have = (5 × 4 + 1) Last day of 4th century is Sunday
= 21 odd days
= 0 odd days
Since the order keeps on cycling, we see that the lat day of the century cannot be Tuesday,
Thursday or Saturday.
Tables : For calculating odd days

Month Jan Feb March April May June July Aug Sep Oct Nov Dec
Odd 3 0/1 3 2 3 2 3 3 2 3 2 3
days ord./Leap
yr

Months of Ist three months 1 IInd Three IIIrd three Ivth (last) three Total year 1
years Jan to 31 March months 1 Apr months 1 July months Jan to 31 Dec.
to 30 June to 30 Sep.

Total days 90 / 91 ord / leap 91 92 92 365/366


ord./leap
Odd days 6 / 0 ord./leap 0 odd day 1 odd day 1 odd day 1 / 2 odd days

89
PRACTICE EXERCISE

1. Find the day of the week on 26 January, 1950.


(A) Tuesday (B) Friday (C) Wednesday (D) Thursday

2. Which two months in a year have the same calendar ?


(A) June, October (B) April, November (C) April, July (D) October, December

3. Are the years 900 and 1000 leap years ?


(A) Yes (B) No (C) Can’t say (D) None of these

4. If it was Saturday on 17th November, 1962 what will be the day on 22nd November, 1964 ?
(A) Monday (B) Tuesday (C) Wednesday (D) Sunday

5. Sangeeta remembers that her father, birthday was certainly after eighth but before thirteenth of
December. Her sister Natasha remembers that their father’s birthday was definitely after ninth but
before fourteenth of December. On which date of December was their father’s birthday ?
(A) 10th (B) 11th (C) 12th (D) Data inadequate

6. Find the day of the week on 15 August, 1947.


(A) Tuesday (B) Friday (C) Wednesday (D) Thursday

7. Karan was born on Saturday 22nd March 1982. On what day of the week was he 14 years 7
months and 8 days of age ?
(A) Sunday (B) Tuesday (C) Wednesday (D) Monday

8. If on 14 day after 5th march be Wednesday, what day of the week will fall on 10th Dec. of the
same year ?
(A) Friday (B) Wednesday (C) Thursday (D) Tuesday

9. If the day before yesterday was Saturday, what day will fall on the day after tomorrow ?
(A) Friday (B) Thursday (C) Wednesday (D) Tuesday

10. If February 1, 1996 is Wednesday, what ay is March 10, 1996 ?


(A) Monday (B) Sunday (C) Saturday (D) Friday

90
11. If the seventh day of a month is three days earlier than Friday, what day will it be on the
nineteenth day of the month ?
(A) Sunday (B) Monday (C) Wednesday (D) Friday

12. Mohini went to the moves nine days ago. She goes to the moves only on Thursday. What day of
the week is today ?
(A) Thursday (B) Saturday (C) Sunday (D) Tuesday

ANSWERS

Que. 1 2 3 4 5 6 7 8 9 10 11 12
Ans. D C B D D B C B C C A B

91
CLOCK TEST

IMPORTANT NOTES :
(i) Minutes hand and hour hand coincides once in every hour. They coincide 11 times in 12
hours & 22 times in 24 hours. They don’t coincide between 12 & 1 O’ Clock.
(ii) Minute hand & hour hand are opposite once in every hour. Then the two hands are opposite
in direction, distance between them is of 30 minutes. They make an angle of 1800 with each
other. They do it 11 times in 12 hours & 22 times in 24 hours. It doesn’t happen between 6 to
7 o’clock.
(iii) Both hands (Minutes & hour) are perpendicular twice in every hour. They make an angle of
90. 22 times in 12 hours and 44 times in 24 hours.
(iv) In One Minute, hour hand moves 1/20 & Minute hand moves 60. In one hour, hour hand
moves 300 & minute hand moves 3600.
(v) In an hour, minute hand moves 55 minutes ahead of hour hand.

HANDS COINCIDE :
Ex.1 At what time between 3 & 4 will the two hands coincide ?
Sol. At 3 o’clock the distance between the two hands is 15 minutes. When they are at zero minutes
distance, they are coincide to each other. The time taken = 15 minutes.
∴ minute hand is 55 minutes ahead of hour hand in 60 minutes.

60
__________1 minutes ________
55
60 × 15 4
_________15 minutes _________ = 3 & 16 min ute
55 11

HANDS ARE OPPOSITE :

Ex2. At what time between 2 & 3 will the two hands are opposite ?
Sol. At 2 o’ clock the distance between the two hands is 10 minutes. When they are at 30 minutes
distance, they are opposite to each other. The time taken (30 + 10) = 4 minutes
∴ minute had is 55 minutes ahead of hour hand in 60 minutes.
60
___________ 1 minute ___________
55
60 × 40 7
__________40 minutes ____________ = 2 & 43 min ute
55 11

92
Table → Hands are opposite
1&2 2&3 3&4 4&5 5&6 6&7 7&8 8&9 9 & 10 1 & 11 11 & 12 12 & 1

6 6
2 7 1 6 5 10 4 9 3 8
1 & 38 2 & 43 3 & 49 4 & 54 7&5 8 & 10 8 & 16 10 & 21 11 & 27 12 & 32
11 11 11 11 11 11 11 11 11 11

HANDS ARE PERPENDICULAR :

Ex.3 At what time between 4 & 5 will the hands are perpendicular ?
Sol. At 4 o’clock the distance between the two hands is 20 minutes. When they are at 15 minutes
distance, they are perpendicular to each other. The time taken 5 and (5 + 30) = 5 and 35 minutes.
∴ minutes hand is 55 minutes ahead of hour hand in 60 minutes.
60
____________1 minutes __________
55
60 × 5 5
___________5 minutes ___________ = 4 & 5 min ute
55 11
60 × 35 2
And _________35 minutes ________ = 4 & 38 min ute
55 11

MIRROR IMAGE OF CLOCK :

1. If the time is between 1 to 11 o; clock, then to find the mirror image, time is subtracted by 11 : 60.
2. If the time is between 11 to 1, then to find the mirror image, time is subtracted by 23 : 60

Ex.4 The time in the clock is 4 : 46, what is the mirror image ?
Sol. 11 : 60 - 4 : 46 = 7 : 14
or
12 - 4 : 46 = 7 : 14

Ex.5 The time in the clock is 12 : 35, then its mirror image will be -
Sol. 23 : 60 - 12 : 35
= 11 : 25

TO FIND THE ANGLE BETWEEN TWO HANDS :


Minute hand moves 300 in 5 minutes & 3600 in 1 hour i.e., It moves 60 in One Minute

Hour Hand moves 300 in 60 minutes

In one minute, it moves 0.50

93
Angle are of two types :
Positive angle : It is obtained by moving from hour hand to minute Hand.
Negative angle : It is obtained by moving from minute hand to hour hand.

Note : Both type of angles are 360 in total. If one angle is known, other can be obtained by

subtracting from 3600

1
Formula : minutes of the given time are multiplied by 5 …… a, Hour hand is multiplied by 30
2
..... b, then a - b is the angle required.

Ex.6 AT 4 : 30, what is the angle formed between hour hand & minute Hand ?
Sol. 4 30 = 165
x5 120
X30 1 450(an)
s

Ex.7 At 3 : 15 what is the angle formed between hour hand & minute hand ?
3

× 30 15
1 1
90 ×5 = 82 
2 2

( −)90
Sol.
1
−7 
2
1 1
360 − 7  = 352 
2 2

PRACTICE EXERCISE

1. At what time are the hand of a clock together between 5 and 6 ?


3 3 3 3
(A) 33 min. past 5 (B) 28 min. pat 5 (C) 27 min. past 5 (D) 26 min. past 5
11 11 11 11
2. At what time between 9 and 10 will the hands of a clock be in the straight line, but not together ?
4
(A) 16 minutes past 9 (B) 16 minutes past 9
11
6 9
(C) 16 minutes past 9 (D) 16 minutes past 9
11 11

94
3. At what time between 5 & 5 : 30 will be hands of a clock be at right angle ?
10 5
(A) 10 minutes past 5 (B) 11 minutes past 5
11 11
10 9
(C) 9 minutes past 5 (D) 10 minutes past 5
11 11

4. Ajay left home for the bus stop 15 minutes earlier than usual. It takes 10 minutes to reach the
stop. He reached the stop at 8.40 a.m. What time does he usually leave home for the bus stop ?
(A) 8.30 a.m. (B) 4.85 a.m. (C) 8.55 a.m. (D) Data inadequate

5. The priest told the devotee, “The temple bell is rung at regular intervals of 45 minutes. The last
bell was rung five minutes ago. The next bell is due to be rung at 7.45 a.m. “At what time did the
priest give this information to the devotee ?
(A) 7.40 a.m. (B) 7.05 a.m. (C) 6.55 a.m. (D) none of these

6. There are twenty people working in an office. The first group of five works between 8.00 A.M. and

2.00 P.M. The second group of ten works between 100 A.M. and 4.00 P.M. And the third group of
five works between 12 noon and 6.00 P.<. There are three computer in the office which all the
employees frequently use. During which of the following hours the computers are likely to be
used most ?
(A) 10.00 A.M. - 12 noon (B) 12 noon - 2.00 P.M.
(C) 1.00 P.M. - 3.00 P.M. (D) 2.00 P.M. - 4.00 P.M.

7. A tired worker slept at 7.45 p.m. If he rose at 12 noon, for how many hours did he sleep ?
(A) 5 hours 15 min. (B) 16 hours 15 min. (C) 12 hours (D) 6 hours 45 min.

8. How many time are the hands of a clocks perpendicular in a day ?


(A) 42 (B) 48 (C) 44 (D) 46

9. If a clock shows 04 : 28 then its mirror image will be ?


(A) 07 : 42 (B) 07 : 32 (C) 08 : 32 (D) 08 : 42

10. A watch, which gain uniformly, is 3 minutes slow at noon an Monday and is 3 minutes 48 seconds
fast at 2 p.m. on the following Monday. What time it was correct ?
(A) 2 p.m. On Tuesday (B) 2 p.m. On Wednesday
(C) 3 p.m. On Thursday (D) 1 p.m. On Friday.

95
11. How many times are the hands of a clocks coincide in a day ?
(A) 10 (B) 11 (C) 12 (D) 22

12. At what time between 2 and 3 O’clock the hands of a clock will make an angle of 1600 ?
(A) 20 minutes past 2 (B) 30 minutes past 2 (C) 40 minutes past 2 (D) 50 minutes past 2

13. Ashish leaves his house at 20 minutes to seven in the morning, reaches Kunal’s house in 25
minutes, they finish their breakfast in another 15 minutes and leave for their office which takes
another 35 minutes. At what time do they leave Kunal’s house to each their office ?
(A) 7.40 am (B) 7.20 am (C) 7.45 cm (D) 8.15 cm

14. The train for Lucknow leaves every two and a half hours from New Delhi Railway Station. An
announcement was made at the station that the train for Lucknow had left 40 minutes ago and the
next train will leave at 18.00 hrs. At what time was the announcement made ?
(A) 15.30 hrs (B) 17.10 hrs (C) 16.00 hrs (D) None of these

15. A monkey climbs 30 feet at the beginning of each hour and rests for a while when he slips back
20 feet before he again starts climbing in the beginning of the next hour. If he begins his ascent at
8.00 a.m., at what time will he first touch a flag at 120 feet from the ground ?
(A) 4 p.m. (B) 5 p.m. (C) 6 p.m. (D) None of these

16. If the two incorrect watches are set at 12 : 00 noon at correct time, when will both the watches
show the correct time for the first time given that the fist watch gains 1 min in 1 hour and second
watch loses 4 min in 2 hours :
(A) 6 pm, 25 days later (B) 12 : 00 noon, 30 days later
(C) 12 noon, 15 days later (D) 6 cm 45 days later

17. Rajeev and Sanjeev are too close friends Rajeev’s watch gains 1 minute in an hour and Sanjeev’s
watch loses 2 minutes in an hour. Once they set both the watches at 12 : 00 noon, with my correct
watch. When will the two incorrect watches of Rajeev and Sanjeev show the same time together ?
(A) 8 days later (B) 10 days later (C) 6 days later (D) can’t be determined

18. Ramu purchased a second hand Swiss watch which is very costly. In this watch the minute-hand
3
and hour hand coincide after every 65 minutes. How much time does the watch lose or gain
11
per day ?
(A) 4 min (B) 5 min (C) 4 min, 20 sec (D) none of these

96
Directions : (19 to 20) : A 12 dial clock has its minute had defective. Whenever it touches dial 12, it
immediately falls down to 6 instead of running smoothly (the hour hand remains
unaffected during that fall). It was set right at 12 ‘O’ clock in the noon.

19. What was the actual time when the minute hand of the clock touched dial 9 for the 5th time ?
(A) 2 : 15 (B) 3 : 00 (C) 5 : 15 (C) 6 : 45

20. If the actual time is 10 : 10, what is the position of the hour hand in that defective clock ?
(A) Between 2 and 3 (B) Between 4 and 5 (C) Between 10 and 11 (D) Between 3 an d 4

ANSWERS

Que. 1 2 3 4 5 6 7 8 9 10
Ans. C B A B B B B C B C
Que. 11 12 13 14 15 16 17 18 19 20
Ans. D C B D C B B A A C

97
CUBES

CUBES :
A cube is three dimensional figure, having 8 corners, 6 surfaces and 12 edges. If a cube is
painted on all of its surfaces with any colour and further divided into various smaller cubes, we get
following results. Smaller cubes with three surfaces painted will be present on the corners of the
big cube.

Smaller cubes with two surface painted will be present on the edges of the big cube. Smaller
cubes with one surface painted will be present on the surfaces of the big cube. Smaller cubes with
no surface painted will be present inside the big cube.

If a cube is painted on all of its surfaces with a colour and then divided into smaller cubes of equal
size then after separation, number of smaller cubes so obtained will be calculated as under :
Number of smaller cubes with three surfaces painted = 8

Number of smaller cubes with two surfaces painted = (n - 2)2 × 12

Number of smaller cubes with one surfaces painted = (n - 2)2 × 6

Number of smaller cubes with no surfaces painted = (n - 2)3


Where n = No of division on the surfaces of the bigger cube

length edge of big cube


=
length of edge of one smaller cube

98
TYPE I :
If a cube is painted on all of its surfaces with single colour and then divided into various smaller
cubes of equal size.

Directions : A cube of side 4 cm. is painted black on all of its surfaces and then divided into
various smaller cubes of side 1 cm each. The smaller cubes so obtained are separated.
4× 4× 4
Total cubes of obtained = = 64
1× 1× 1
side of big cube 4
Here n= = =4
side of small cube 1

1. Number of smaller cubes with three surfaces painted =8

2. Number of smaller cubes with two surfaces painted = (n - 2) × 12


= (4 - 2) × 12 = 24

3. Number of smaller cubes with one surfaces painted = (n - 2)2 × 6

= (4 - 2)2 × 6 = 4 × 6 = 24

4. Number of smaller cubes with no surface painted = (n - 2)3 = (4 - 2)3 = (2)3 = 8

TYPE II :
If a cube is painted on all of its surfaces with different colours and then divided into various
smaller cubes of equal size.

Directions : A cube of side 4 cm is painted black on the pair of one opposite surfaces, blue, on the
pair of another opposite surfaces and red on remaining pair of opposite surfaces, the cube
is now divided into smaller cube of equal side of 1 cm each.

1. Number of smaller cubes with three surfaces painted = 8


(These smaller cubes will have all three surfaces painted with different colour blue, black and

red.)

99
2. Number of smaller cubes with two surfaces painted = 24. And out of this -
(a) Number of cubes with two surfaces painted with black and blue colour = 8.
(b) Number of cubes with two surfaces painted with blue and red colour = 8.
(c) Number of cubes with two surfaces painted with black and red colour = 8.

3. Number of smaller cubes with one surface painted = 24. And out of this -
(a) Number of cubes with one surface painted with black colour = 8.
(b) Number of cubes with one surface painted with blue colour = 8.
(c) Number of cubes with one surface painted with red colour = 8.

TYPE III :
If a cube is painted on its surfaces in such a way that one pair of opposite surfaces is left
unpainted.

Directions : A cube of side 4 cm is painted red on the pair of one opposite surfaces, green on the
pair of another opposite surfaces and one pair of opposite surfaces is left unpainted. Now
the cube is divided into 64 smaller cubes of side 1 cm each.

1. Number of smaller cubes with three surfaces painted = 0 (Because each smaller cube at the
corner is attached to a surface which is unpainted.)

2. Number of smaller cubes with two surfaces painted = Number of cubes present at the corners +
Numbers of cubes present at 4 edges
= 8 + (n - 2) × 4
= 8 + 8 = 16

3. Number of smaller cubes with one surface painted.


= Number of cubes present at the 8 edges + number of cubes present at the four surfaces.
= (n - 2) × 8 + (n - 2)2 × 4
= ( 2 × 8 + 4 × 4 = 16 + 16 = 32

4. Number of smaller cubes with no side painted


= Number of cube on the two unpainted surfaces + number of cubes present inside the cube.
= (n - 2)2 × 2 + (n - 2)3
= 4 × 2 + (2)3
= 8 + 8 = 16

100
TYPE IV:
If a cube is painted on its surfaces in such a way that one pair of adjacent surfaces is left
unpainted/

Directions : A cube of side 4 cm is painted red on the pair of one adjacent surfaces, green on the
pair of other adjacent surfaces and two adjacent surfaces are left unpainted. Now the cube
is divided into 64 smaller cubes of side 1 cm each.

1. Number of smaller cubes with three surfaces painted = Number of smaller cubes at two
corners = 2

2. Number of smaller cubes with two surfaces painted = Number of smaller cubes at four corners +
Number of smaller cubes at 5 edges.
= 4 + (n - 2) × 5 = 4 + 2 × 5
=4 + 10 = 14

3. Number of smaller cubes with one surface painted = Number of smaller cubes at four surfaces +
Number of smaller cubes at 6 edges + Number of smaller cubes at two corners.

= (n - 2)2 × 4 + (n - 2) × 6 + 2
= 4 × 4 + 2 × + 2 = 16 + 12 = 28 + 2 = 30

4. Number of smaller cubes with no surfaces painted = Number of smaller cubes from inside the big
cube + Number of cubes at two surfaces + Numbers of cubes at one edge.

= (n - 2)2 + (n - 2)2 × 2 + (n - 2)

= (2)2 + (2)2 × + 2
= 8 + 8 + 2 = 18

101
PRACTICE EXERCISE

Directions : (1 to 5) A cube is coloured orange on one face, pink on the opposite face, brown
on one face and silver on a face adjacent to the brown face. The other two faces are left
uncoloured. It is then cut into 125 smaller cubes of equal size. Now answer the following
questions based on the above statements

1. How many cubes have at least one face coloured pink ?


(A) 1 (B) 9 (C) 16 (D) 25

2. How many cubes have all the faces uncloured ?


(A) 24 (B) 36 (C) 48 (D) 64

3. How many cubes have at least two faces coloured ?


(A) 19 (B) 20 (C) 21 (D) 23

4. How many cubes are coloured orange on one face and have the remaining faces uncoloured ?
(A) 8 (B) 12 (C) 14 (D) 16

5. How many cubes one coloured silver on one face, orange or pink on another face and have four
uncoloured faces ?
(A) 8 (B) 10 (C) 12 (D) 16

Directions : (6 to 11) A cube is painted red on two adjacent surfaces and black on the surfaces
opposite to red surfaces and green on the remaining faces. Now the cube is cut into sixty
four smaller cubes of equal size.

6. How many smaller cubes have only one surface painted ?


(A) 8 (B) 16 (C) 24 (D) 32

7. How many smaller cubes will have to surface painted ?


(A) 0 (B) 4 (C) 8 (D) 16

8. How many smaller cubes have less than three surfaces painted ?
(A) 8 (B) 24 (C) 28 (D) 48

102
9. How many smaller cubes have three surfaces painted ?
(A) 4 (B) 8 (C) 16 (D) 24

10. How many smaller cubes with two surfaces painted have one face green and one of the adjacent
faces black or red ?
(A) 8 (B) 16 (C) 24 (D) 28

11. How many smaller cubes have at least one surface painted with green colour ?
() 8 (B) 24 (C) 32 (D) 56

Directions : (12 to 16) A cube of 4 cm has been painted on its surfaces is such a way that two
opposite surfaces have been painted blue and two adjacent surfaces have been painted
red. Two remaining surfaces have been left unpainted. Now the cube is cut into smaller
cubes of side 1 cm each.

12. How many cubes will have no side painted ?


(A) 18 (B) 16 (C) 22 (D) 8

13. How many cubes will have at least red colour on its surfaces ?
(A) 20 (B) 22 (C) 28 (D) 32

14. How many cubes will have at least blue colour on its surfaces ?
(A) 20 (B) 8 (C) 24 (D) 32

15. How many cubes will have only two surfaces painted with red and blue colour respectively ?
(A) 8 (B) 12 (C) 24 (D) 30

16. How many cubes will have three surfaces coloured ?


(A) 3 (B) 4 (C) 2 (D) 16

Directions : (17 to 26) A solid cube has been painted yellow, blue and black on pairs of opposite
faces. The cube in then cut into 36 smaller cubes such that 32 cubes are of the same size
while 4 others are of bigger sizes. Also no faces of any of the bigger cubes is painted blue.

17. How many cubes have at least one face painted blue ?
(A) 0 (B) 8 (C) 16 (D) 32

18. How many cubes have only one faces painted ?


(A) 24 (B) 20 (C) 8 (D) 12

103
19. How many cubes have only two faces painted ?
(A) 24 (B) 20 (C) 16 (D) 8

20. How many cubes have at least two faces painted ?


(A) 36 (B) 34 (C) 28 (D) 24

21. How many cubes have only thee faces painted ?


(A) 8 (B) 4 (C) 2 (D) 0

22. How many cubes do not have any of their faces painted yellow ?
(A) 0 (B) 4 (C) 8 (D) 16

23. How many cubes have at least one of their faces painted black ?
(A) 0 (B) 8 (C) 16 (D) 20

24. How many cubes have at least one of their faces painted yellow or blue ?
(A) 36 (B) 32 (C) 16 (D) 0

25. How many cubes have no face painted ?


(A) 8 (B) 4 (C) 1 (D) 0

26. How many cubes have two faces painted yellow and black respectively ?
(A) 0 (B) 8 (C) 12 (D) 16

Directions : (27 to 31) A cuboids of dimensions (6 cm × 4 cm × 1 cm) is painted black on both the
surfaces of dimensions (4 cm × 1 cm), green on the surfaces of dimensions (6 cm × 4 cm).
and red on the surfaces of dimensions (6 cm × 1 cm). Now the block is divided into various
smaller cubes of side 1 cm, each. The smaller cubes so obtained are separated.

27. How many cubes will have all three colours black, green and red each at least on one side ?
(A) 16 (B) 12 (C) 10 (D) 8

28. How many cubes will be formed ?


(A) 6 (B) 12 (C) 16 (D) 24

29. If cubes having only black as well as green colour are removed then how many cubes will be left ?
(A) 4 (B) 8 (C) 16 (D) 30

30. How many cubes will have 4 coloured sides and 2 sides without colour ?

104
(A) 8 (B) 4 (C) 16 (D) 10

31. How many cubes will have two sides with green colour and remaining sides without any colour ?
(A) 12 (B) 10 (C) 8 (D) 4

ANSWERS

Que. 1 2 3 4 5 6 7 8 9 10 11
Ans. D C C D A C C D B B C
Que. 12 13 14 15 16 17 18 19 20 21 22
Ans. A C D B C D C B C A D
Que. 23 24 25 26 27 28 29 30 31
Ans. D A D C A D C B C

105
DICES

CATEGORY I :
Ex. A dice has been thrown four times and produces following results.

(i) (ii) (iii) (iv)

Which number will appear opposite to the number 3 ?


(A) 4 (B) 5 (C) 6 (D) 1

Sol. From the figures (i), (ii) and (iv) we find that numbers 6, 1, 5 and 2 appear on the adjacent
surfaces to the number 3. Therefore, number 4 will be opposite to number 3. Hence option (a) is
the answer.

CATEGORY II :

Ex. The figures given below show the two different position of a dive. Which number will appear
opposite to number 2.

(i) (ii)
(A) 3 (B) 4 (C) 5 (D) 6

Sol. The above question, where only two positions of a dive are given, can easily be solved with the
following method.

Step I.
The dice, when unfolded, will appear as shown in the figure given on the right side.
Step II.
Write the common number to both the dice in the middle block. Since common number is 4,
hence number 4 will appear in the central block.

106
Step III.
Consider the figure (i) and write the first number in the anti-clockwise direction of number 4,
(common number) in block I and second number in block II. Therefore, numbers 3 and 2 being the
first and second number to 4 in anticlockwise directions respectively will appear in block I & II
respectively.

Step IV.
Consider figure (ii) and wire first and second number in the anticlock-wise direction to number 4,
(common number) in block (iii) & (IV). Hence number 6 and 5 will appear in the blocks III and IV
respectively.

Step V.
Write remaining number in the remaining block. Therefore, number 1 will come in the remaining
block. Now, from the unfolded figures we find that number opposite to 6 is 3, number opposite to 2
is 5 and number opposite to 4 1. Therefore, option (c) is our answer

CATEGORY III :
Ex. From the following figures of dice, find which number will come in place of ‘?’

(i) (ii) (iii)


(A) 4 (B) 5 (C) 2 (D) 3
Sol. If the above dice is unfolded, it will look like as the figure (1) given below.

Figur (1)
Now the number in place of ‘?’ can be obtained by making a slight change in the figure as given
here. Now comparing figure (2) with figure (iii) as above, we et that number in place of ? is 3.

Figur (2)

107
CATEGORY IV :
Ex. Which of the following dices is identical to the unfolded figure as shown here ?

(A) (B) (C) (D)

Sol. From the unfolded figure of dice, we find that number opposite to 2 is 4, for 5 it is 3 and for 1 it is
6. From this result we can definitely say that figure (B), (C) and (D) can not be the answer figures
as number lying on the opposite pair of surfaces are present on the adjacent surfaces. Hence fig.
(A) is our answer.

PRACTICE EXERCISE

1. Which alphabet is opposite D ?

(i) (ii)
(A) E (B) C (C) F (D) A

2. What should be the number opposite 4 ?

(i) (ii) (iii)


(A) 5 (B) 1 (C) 3 (D) 2

3.

(i) (ii) (iii) (iv)


Which letter will be opposite to letter D ?
(A) A (B) B (C) E (D) F

108
Directions : (4 to 5) The figure (X) given below is the unfolded position of a cubical dice. In each of
the following questions this unfolded figure is followed by four different figures of dice.
You have to select the figure which is identical to the figure (X).

4. (X)

(A) (B) (C) (D)

5. (X)

(A) (B) (C) (D)

Directions (6 to 8 ) In each of the following questions, select the correct option for the question
asked.

(i) (ii)

6. Which number will come opposite to number ?


(A) 5 (B) 1 (C) 6 (D) 3

7. Which number will come opposite to number 6 ?


(A) 1 (B) 5 (C) 4 (D) 3

8. Which number will come opposite to number 4 ?


(A) 3 (B) 5 (C) 1 (D) 2

109
9. On the basis of two figures of dice, you have to tell what number will be on the opposite face of
number 5 ?

(i) (ii)
(A) 1 (B) 2 (C) 4 (D) 6

10. Which symbol will appear on the opposite surface to the symbol x ?

(i) (ii)
(A) ÷ (B) = (X) ∆ (D) O

11. Three positions of the same dice are given below. Observe the figures carefully and tell which
number will come in place of ‘?’

(i) (ii) (iii)


(A) 1 (B) 6 (C) 3 (D) 5

12. On the basis of the following figures you have to tell which number will come in place of ‘?’

(i) (ii) (iii)


(A) 2 (B) 3 (C) 6 (D) 4
Directions : (13 to 15) Choose from the alternatives, the boxes that will be formed when figure (X)
is folded :

13. (X)

(A) (B) (C) (D)

110
14. (X)

(A) (B) (C) (D)

15. (x)

(A) (B) (C) (D)

Directions : (16) The six faces of a cube have been marked with number 1, 2, 3, 4, 5 and 6
respectively. This cube is rolled down three times. The three positions are given. Choose
the figure that will be formed when the cube is unfolded.

16.

(A) (B) (C) (D)

17. Which number is opposite 3 in a standard dive given below ?

(A) 1 (B) 4 (C) 5 (D) Can’t be determined

111
18. Which number is opposite 4 ?

(i) (ii)

(A) 5 (B) 3 (C) 2 (D) 1

Directions : (19) In the following questions four positions of the same dice have been shown. You
have to seen these figures and select the number opposite to the number as asked in each
question.

19.

(i) (ii) (iii) (iv)


Which number is opposite to number 5 ?
(A) 6 (B) 5 (C) 1 (D) 3

Directions : (20 to 23) In each of the following questions, a diagram has been given which can be
folded into a cube. the entries given in the squares indicate the entries on the face of the
cube. In each questions a number or a letter has been given on the left. Of the four
alternatives given against it, you have to find the one that would appear on the face
opposite to it in the cube.

20. Which letter is opposite Q ?

(A) L (B) M (C) N (D) P

21. Which number/letter is opposite 2 ?

(A) A (B) C (C) 1 (D) 3

112
22. Which number/letter is opposite O ?

(A) L (B) M (C) N (D) 2

23. Which letter is opposite R ?

(A) P (B) S (C) T (D) U

ANSWERS

ANSWER KEY
Que. 1 2 3 4 5 6 7 8 9 10 11 12
Ans. B B A D B D A B C D A B
Que. 13 14 15 16 17 18 19 20 21 22 23
Ans. D B D C B A C C A B B

113
FIGURE PARTITION

FIGURE PARTITION :
The chapter on Analytical Reasoning involves the problems relating to the counting of geometrical
figures in a given complex figure. The systematic method for determining the number of any
particular type of figure by the analysis of the complex figure would be clear from the examples
that follow.

Ex.1 What is the number of straight lines in the following figure ?


(A) 11
(B) 14
(C) 16
(D) 17
Sol. (B) The figure is labelled as shown.
Clearly, there are 3 horizontal liens namely AE, LF and KG.
There are 5 vertical lines : AL, BJ, CI, DH and EG.
There are 6 slanting lines : LC, KE, IF, LI, AG and CF.
Thus, there are 3 + 5 + 6 = 14 straight lines in the figure.

Ex.2 How many squares does the figure have?


(A) 6
(B) 7
(C) 9
(D) 10
Sol. The figure may be labelled as shown :
The squares composed to two components each, are ABKJ,
BCLK, CDEL, LEFG, KLGH and JKHI. Thus, there are 6 such
squares. Only one square, KCEG is composed of four
components. Two squares namely, ACGI and BDFH are
composed of eight components each. Thus, there are 2 such
squares.
∴ There are 6 + 1 + 2 = 9 squares in the given figure.

114
Ex.3 How many parallelograms are there in the figure below ?
(A) 14 (B) 15
(C) 16 (D) 18

Sol. We can label the figure as shown.


The simplest parallelograms are ABFE, BCGF, CDHG, EFJI,
FGKJ and GHKL. These are 6 in number. The parallelograms
composed of two components each, are ACGE, BDHF, EGKI,
FHLJ, ABJI and CDLK. Thus, there are 7 such parallelograms.
The parallelograms composed of four components each, are
ACKI and BDLJ. i.e. 2 in number. There is only one
parallelograms composed of six components, namely, ADLI.
Thus there are 6 + 7 + 2 + 1 = 16 parallelograms in the figure.
Hence,

Ex.4 What is the number of rectangles in the following figure ?


(A) 6
(B) 7
(C) 8
(D) 9

Sol. The figure is labelled as shown :


Simplest rectangles are AEHG, EFIH, FBKJ, JKCL and GILD.
i.e. there are 5 such rectangles. The rectangles composed of
two components each are AFIG and FBCL. Thus, there are 2
such rectangles. Only one rectangles, namely AFLD is
composed of 3 components and only one rectangle, namely
ABCD is composed of 5 components. Thus, there are 5 + 2 + 1
+ 1 = 9 rectangles in the figure.

Ex.5 Determine the number of pentagons in the following figure :


(A) 5
(B) 6
(C) 8
(D) 10

Sol. The figure may be labelled as follows :


In this case, six pentagons have been formed by the
combination of three triangles and two rhombuses - ADFHJ,
CFHJL, EHJLB, GJLBD, ILBDF and KBDFH. Four other
pentagons are formed by the combination of three triangles and
one rhombus - LCFHM, LBEHM, BKFHM and BLIFM. Thus,
there are 10 pentagons in the figure.

115
NON-VERBAL ANALOGY &
CLASSIFICATION

ANALOGY :
In this type, the problem Figures are presented in two units. The first unit contains two figures,
one in each square. The second unit contains one figure in the first square and a questions mark
(?) in the second you have to find out from among the figures A,B,C,D and E as to which one
should replace the questions mark after finding the relationship between the two figures in the first
unit of the problem figure. Some examples are discussed below.

Ex.1 Problem Figures Answer Figures

P1 P2 P3 P4 (A) (B) (C) (E) (E)

Sol. (C) P2 contains the lower half of P1. Hence, answer figure C replaces the question mark.

Ex.2 Problem Figures Answer Figures

P1 P2 P3 P4 (A) (B) (C) (D) (E)

Sol. (E) The lower LHS figure of portion in P1 becomes the upper portion in P2, shifted to the other
side. Similarly RHS figure of the upper portion in P1 becomes the lower portion P2 shifted to the
other side with one vertical line therein. The other two halves are deleted.

Ex.3 Problem Figures Answer Figures

P1 P2 P3 P4 (A) (B) (C) (E) (E)


Sol. (A) P1 is rotated 1800 ACW or CW to obtain P2. Then the shaded and the unshaded leaves are
interchanged. Hence, (A) should replace the questions mark.
CLASSIFICATION :

116
In these type of questions, five figures numbered (A), (B), (C), (D) and (E) are given. These are
treated both problem Figures as well as the Answer figures. Four out of these five figures are
related to each other by way of having some common characteristics ans so form a group. Out of
these five, you have to identify one figure which does not belong to group.

Ex.4

(A) (B) (C) (D) (E)


Sol. (C) Both the arrow heads are in the same direction in figure (C). In all other figures, they are in the
opposite direction, Hence , (C) is the answer

Ex.5

(A) (B) (C) (D) (E)


Sol. (E) Between the shaded portion and the arrow, there are two triangles in figure (A), (B), (C) and
(D).

Ex.6

(A) (B) (C) (D) (E)


Sol. (C) Figures (A) and (D) form a group. The bars are interchanged here. Similarly, figure (B) and
(D). Hence (C) is the odd one out.

PRACTICE EXERCISE

Directions : (1 to 11) Figures 1’ and 2 are related in a particular manner. Establish the same
relationship between figures 3 and 4 by choosing a figure from amongst the five
alternatives, which would replace the questions mark in figure (4).
Problem Figures Answer Figures

1.

(1) (2) (3) (4) (A) (B) (C) (D) (E)

2.

(1) (2) (3) (4) (A) (B) (C) (D) (E)

3.

(1) (2) (3) (4) (A) (B) (C) (D) (E)

117
4.

(1) (2) (3) (4) (A) (B) (C) (D) (E)

5.

(1) (2) (3) (4) (A) (B) (C) (D) (E)

6.

(1) (2) (3) (4) (A) (B) (C) (D) (E)

7.

(1) (2) (3) (4) (A) (B) (C) (D) (E)

8.

(1) (2) (3) (4) (A) (B) (C) (D) (E)

9.

(1) (2) (3) (4) (A) (B) (C) (D) (E)

10.

(1) (2) (3) (4) (A) (B) (C) (D) (E)

11.

(1) (2) (3) (4) (A) (B) (C) (D) (E)

118
Directions : (12 to 16) In each of the following problems, a related pair of figures is followed by
five numbered pairs of figures. Select the pair that has a relationship similar to that in the
original pair.

12.

(x) (A) (B) (C) (D) (E)

13.

(x) (A) (B) (C) (D) (E)

14.

(x) (A) (B) (C) (D) (E)

15.

(x) (A) (B) (C) (D) (E)

16.

(x) (A) (B) (C) (D) (E)

Directions : (17 to 21) In each of the following questions, in four out of the given five pairs of
figures, the first element is related to the second element in the same particular manner.
Spot out the pair in which this relationship does not exist between the figures.

17.

(A) (B) (C) (D) (E)

18.

(A) (B) (C) (D) (E)

119
19.

(A) (B) (C) (D) (E)

20.

(A) (B) (C) (D) (E)

21.

(A) (B) (C) (D) (E)

Directions : (22 to 26) Each of the following questions bears four figure numbered 1,2,3 and 4
which constitute the Problem Set and four other figures numbered A,B,C and D which
continue the Answer Set. Figures 1 and 2 are related in a particular way/ Establish a similar
relationship between figures 3 and 4 by choosing a figure from the Answer set that would
best substitute figure (4) in the Problem set. In case if none of the figures of the Answer set
is suitable then answer is E.

Problem Answer Figures

22.

(1) (2) (3) (4) (A) (B) (C) (D)

23.

(1) (2) (3) (4) (A) (B) (C) (D)

24.

(1) (2) (3) (4) (A) (B) (C) (D)

25.

(1) (2) (3) (4) (A) (B) (C) (D)

120
26.

(1) (2) (3) (4) (A) (B) (C) (D)

Directions: (27 to 41) Out of the five figures (A), (B), (C), (D) and (E), given in each problem, four
are similar in a certain way. However, one figure is not like the other four. Choose the
figure which is different from the rest.

27.

(A) (B) (C) (D) (E)

28.

(A) (B) (C) (D) (E)

29.

(A) (B) (C) (D) (E)

30.

(A) (B) (C) (D) (E)

31.

(A) (B) (C) (D) (E)

32.

(A) (B) (C) (D) (E)

33.

(A) (B) (C) (D) (E)

121
34.

(A) (B) (C) (D) (E)

35.

(A) (B) (C) (D) (E)

36.

(A) (B) (C) (D)

37.

(A) (B) (C) (D) (E)

38.

(A) (B) (C) (D)

39.

(A) (B) (C) (D) (E)

40.

(A) (B) (C) (D) (E)

41.

(A) (B) (C) (D) (E)

122
Directions : (42 to 46) The following problems contain four numbered figures (1,2,3 and 4) forming
the Problem Set and five numbered figures (A,B,C,D & E) forming the Answer Set. The four
Problem figures have certain common features. Select a figure from amongst the Answer
Figures which is similar to the Problem Figures.

Problem Figures Answer Figures

42.

(1) (2) (3) (4) (A) (B) (C) (D) (E)

43.

(1) (2) (3) (4) (A) (B) (C) (D) (E)

44.

(1) (2) (3) (4) (A) (B) (C) (D) (E)

45.

(1) (2) (3) (4) (A) (B) (C) (D) (E)

46.

(1) (2) (3) (4) (A) (B) (C) (D) (E)

ANSWERS

Que. 1 2 3 4 5 6 7 8 9 10 11 12
Ans. C B C C B B E E D A A D
Que. 13 14 15 16 17 18 19 20 21 22 23 24
Ans. A A B C B D D C C D C A
Que. 25 26 27 28 29 30 31 32 33 34 35 36
Ans. D C A D D C D C B E D D
Que. 37 38 39 40 41 42 43 44 45 46
Ans. B A B B E D A D E C

123
PYRAMIDS & MISCELLANEOUS

PYRAMIDS :
Brief review of concepts : The questions in this unit are based on the pyramid of numbers from
1 to 100, we given below.
1
234
98765
10 11 12 1 14 15 16
25 24 23 22 21 20 19 18 17
26 27 28 29 30 31 32 33 34 35 36
49 48 47 46 45 44 43 42 41 40 39 38 37
50 51 52 53 54 55 56 57 58 59 60 61 62 63 64
81 80 79 78 77 76 75 74 73 7 71 70 69 68 67 66 65
82 83 84 85 86 87 88 89 90 91 92 93 94 95 96 97 98 99 100

Many types of questions are possible based on the above pattern from instance, formation of
parallel lines, perpendicular lines, triangles, squares, etc. by taking numbers in order.

Type 1 Fill the blanks from the choice given below.


129 : 145 :: 3811 : ?
(A) 3713 (B) 3289 (C) 346 (D) 3614 (E) 3615
There are two groups of numbers. The number on the right hand side must
have the same relations as the numbers on the left hand side. 129 and 1245, in
the above pyramid, form a pattern.
Hence the number in the blank on the right hand side must form same pattern
with 3811. Therefore, the answer is 3615 which form the pattern.

Type 2 Fill the blank from the choice given below.


2812 : 765 : 91123 : ?

(A) 12110 (B) 121314 (C) 122132 (D) 303132 (E) 122230

The two numbers on the left hand side form perpendicular lines in the pyramid.
Therefore, the numbers on the right hand side must be of the same pattern. The
answer to the above questions should be (B) 121314 to satisfy the same
relation.

124
Type 3 Fill be blanc form the choice given below.
234 : 4614 :: 2812 : ?
(A) 3713 (B) 34614 (C) 121314 (D) 131415 (E) 122239
The numbers on the left-hand side form two side of a square. The
numbers have been chosen in continuity. 2812 and 121314 are in
continuity and form the other two sides of the same square. Hence the
answer is (C).

Type 4 Fill the blank from the choice given below.


507986 : 772112 :: 537689 : ?
(A) 745742 (B) 735841 (C) 716039 (D) 755643 (E) 775543
Hence two numbers on the left-hand side form perpendicular lines.
Therefore, the numbers on the right-hand side must form the same point,
taking numbers in order. Hence the answer is (A) 745742.

Type 5 Fill the blank from the choice given below.


322120 : 321920 : : 324342 : ?
(A) 324142 (B) 324132 (C) 323143 (D) 323319 (E) 324133
The numbers on the left-hand side form a triangle. Therefore, the numbers on the right-hand side
must form a triangle with common point 32. Hence, the answer is (A) 324142.

MISCELLANEOUS :
Ex.1 Select a figure form the four alternatives, which when placed in the blank space of figure (X)
would complete the pattern.

(X) (A) (B) (C) (D)

Sol. (D) Clearly, figure (D) will complete the pattern when placed in the blank space of figure (X) as
shown below.

Ex.2 In the following questions problems, a square transparent sheet with a pattern is given. Figure out
form amongst four alternatives as to how the patter would appear when the transparent sheet is
folded at the dotted line.

(X) (A) (B) (C) (D)

Sol. (B) Clearly, the lower half of the square sheet has been folded over the upper half. Hence, the
bent life in the lower half will be inverted over the other half so that a “V” shaped figure is formed.

125
Ex.3 Consider the following three figures, marked X, Y, Z showing one fold in X, another in Y and cut in
Z. From amongst the answer figures A,B,C and D, select the one, showing the unfolded position
of Z.

X Y Z

(A) (B) (C) (D)

Sol. (C) In figure X, the upper triangular half of the paper has been folded over the lower half. In figure,
Y, the paper is refolded to a quarter triangle. In figure Z, a square has been punched in the folded
paper. Clearly the square will appear in each of the triangular quarters of the paper. Thus, when
the paper is unfolded, four squares will appear symmetrically over it and it will resemble figure (C).

Ex4. In the following question, choose the alternative figure in which the question figure (X) is
embedded.

(X) (A) (B) (C) (D)


Sol. (D) it is clear from the alternative figures that figure (X) is embedded in figure (D)

PRACTICE EXERCISE

Directions : (1 to 12) Fill the blanks in the following questions from the choice given below. Which
are based on pyramids.

1. 192021 : 213241 :: 414243 : ?


(A) 433023 (B) 435871 (C) 414039 (D) 435773

2. 494852 : 818084 :: 101123 : ?


(A) 131221 (B) 494852 (C) 252428 (D) 262747

3. 432946 : 443241 :: ? : 706267


(A) 725675 (B) 695972 (C) 705873 (D) 696366

4. 171210 : 31311 :: 19415858 : ?


(A) 335957 (B) 333342 (C) 333460 (D) 325961

126
5. 151933 : 425774 :: 132131 : ?
(A) 435675 (B) 445576 (C) 455477 (D) 304554

6. 3812 : 3614 :: 284753 : ?


(A) 284654 (B) 282930 (C) 294456 (D) 284555

7. 151835 : 193439 :: 274655 : ?


(A) 274654 (B) 465674 (C) 475475 (D) 455673

8. 324356 : 324160 :: 425774 : ?


(A) 425970 (B) 425971 (C) 423122 (D) 322112

9. 474849 : 495277 :: 777879 : ?


(A) 795149 (B) 795247 (C) 527978 (D) 525377

10. 56443022 : 131415 :: 18344060 : ?


(A) 604132 (B) 595857 (C) 717273 (D) 606162

11. 252845 : 274655 :: 264754 : ?


(A) 102330 (B) 495277 (C) 485376 (D) 262510

12. 767574 : 564430 :: 899091 : ?


(A) 929394 (B) 917357 (C) 765446 (D) 735743

Directions : (13 to 20) Select a figure form the alternative which when placed in the blank space of
(x) would complete the pattern ?

13.

(X) (A) (B) (C) (D)

14.

(X) (A) (B) (C) (D)

15.

(X) (A) (B) (C) (D)

127
16.

(X) (A) (B) (C) (D)

17.

(X) (A) (B) (C) (D)

18.

(X) (A) (B) (C) (D)

19.

(X) (A) (B) (C) (D)

20.

(X) (A) (B) (C) (D)

Direction : (21 to 25) A square transparent sheet with a pattern is given in figure X. Find out form
amongst the alternatives as to how the pattern would appear when the transparent sheet is
folded at the doted line.

21.

(X) (A) (B) (C) (D)

22.

(X) (A) (B) (C) (D)

128
23.

(X) (A) (B) (C) (D)

24.

(X) (A) (B) (C) (D)

25.

(X) (A) (B) (C) (D)

Directions : (26 to 32) A sheet has been folded in the manner as shown in X, Y and Z respectively
and punched. You have to choose from the alternatives how it will look when unfolded.

26.
X Y Z

(A) (B) (C) (D)

27.

X Y Z

(A) (B) (C) (D)

129
28.

X Y Z

(A) (B) (C) (D)

29.

X Y Z

(A) (B) (C) (D)

30.

X Y Z

(A) (B) (C) (D)

31.

X Y Z

(A) (B) (C) (D)

130
32.

X Y Z

(A) (B) (C) (D)

Directions : (33 to 36) In each of the following questions, choose the alternative figure in which
the questions figure (X) is embedded.

33.

(X) (A) (B) (C) (D)

34.

(X) (A) (B) (C) (D)

35.

(X) (A) (B) (C) (D)


36.

(X) (A) (B) (C) (D)

ANSWERS

Que. 1 2 3 4 5 6 7 8 9 10 11 12
Ans. B D B A B D C A B C C D
Que. 13 14 15 16 17 18 19 20 21 22 23 24
Ans. C D B A D A C A D B D A
Que. 25 26 27 28 29 30 31 32 33 34 35 36
Ans. B B B D A A B A B D A D

131

Anda mungkin juga menyukai